PDA

View Full Version : The arguement that recently convinced me of god's existence


Pages : [1] 2

Allinlife
11-21-2005, 06:12 PM
So my close christian friend lends me a book, because I've always been bugging her and questioning christianity, and she told me it'd answer most of my questions.

So I start reading the book and the very first chapter talks about this.

Say you are walking down a field, and you see a wooden chair in middle of no where. Now you know someone must have made that chair, because chair's don't just "happen". You've never witnessed the builder of the chair working on the chair, but you could only assume that someone out there, made the chair and placed it there for whatever reason.

Now if something as simple as a wooden chair can't just "happen" the book argues that something as complicated as humans, cell structures, plants, animals, countless laws of physics that govern the universe could not have just "happened". Somebody must have created us. If you think a wooden chair could not exist with out a builder, consider how infinately more complex body structures we humans have.

And I think the reasoning is fairly solid. I do now think that it is silly to imagine things like mitosis, DNA, human eyes could have appeared by random chance.

The book also goes into absolute lack of evidence in macro-evolution (aka missing links) and how the scientists were still unable to create life out of chemical reactions as they proclaim.

But I do not want to get into the macro evolution/ biogenesis theory stuff, but just would like a discussion on use of logic in the chair anology.

thanks.

hmkpoker
11-21-2005, 06:20 PM
God is infinitely more complex and even less likely to have happened by random chance. Gods don't just "happen." So what created God?

maurile
11-21-2005, 06:26 PM
[ QUOTE ]
The book also goes into absolute lack of evidence in macro-evolution (aka missing links) . . .

[/ QUOTE ]
Sounds like an extremely uninformed book.

Allinlife
11-21-2005, 06:40 PM
[ QUOTE ]
[ QUOTE ]
The book also goes into absolute lack of evidence in macro-evolution (aka missing links) . . .

[/ QUOTE ]
Sounds like an extremely uninformed book.

[/ QUOTE ]
Okay it may be a biased book, but I don't really intend to start another creation vs evolution thing going here...because they always end up in some ridiculous 20 page flame fest.

As for question of "who created god", god is supposed to exist outside time and space, so those things do not bind him. ... a reasonable anology I think is.. assuming we are like ants kept in a 2 dimensional container, we will never understand the world of 3-dimensions. And since time is the 4th dimension, and if god could exist outside much higher dimensions, we will never understand wth is going on there.

could you two PM me any arguements so I could keep topic I want to discuss alone with others that would be intersted in discussing them?

maurile
11-21-2005, 07:13 PM
[ QUOTE ]
Now if something as simple as a wooden chair can't just "happen" the book argues that something as complicated as humans, cell structures, plants, animals, countless laws of physics that govern the universe could not have just "happened".

[/ QUOTE ]
We already know how chairs are made: people craft them out of wood using various tools and stuff.

We also already know how plants are formed. They grow from seeds. Nobody crafts them using any tools or anything.

If we didn't already know these things -- if we truly came across a chair in a desert and we had never seen a chair before and had no clue as to how they were made -- then we would be foolish to conclude that it was crafted by a designer. Maybe it was, or maybe it was formed more like a plant: it somehow grew from a seed. Or maybe there's some other possibility.

So the analogy fails. You can't say "chairs are made by people, therefore plants are made by people." Plants are not made by people.

The fact is that there are several different ways things can end up looking "designed." One is for them to actually have been designed (like a chair). Another is for them to have biologically evolved (like a plant). And importantly, there is no foolproof way of looking at an object we've never seen before and knowing whether it was designed or evolved (or something else).

(There are certain traits we can look for, however, that will give hints. Forms that have biologically evolved are often "designed" rather stupidly in certain ways due to their peculiar evolutionary history. Thus we see whales with remnants of hind limbs, humans with remnants of tail bones, fish that live in pitch-black caves that have non-functioning eyes, etc. A designer would be unlikely to include such useless features, although we can't rule out the possibility that the designer was just really stupid. So as I said, there's no foolproof way for distinguishing between evolved entities and designed entities.)

Trantor
11-21-2005, 07:25 PM
[ QUOTE ]
So my close christian friend lends me a book, because I've always been bugging her and questioning christianity, and she told me it'd answer most of my questions.

So I start reading the book and the very first chapter talks about this.

Say you are walking down a field, and you see a wooden chair in middle of no where. Now you know someone must have made that chair, because chair's don't just "happen". You've never witnessed the builder of the chair working on the chair, but you could only assume that someone out there, made the chair and placed it there for whatever reason.

Now if something as simple as a wooden chair can't just "happen" the book argues that something as complicated as humans, cell structures, plants, animals, countless laws of physics that govern the universe could not have just "happened". Somebody must have created us. If you think a wooden chair could not exist with out a builder, consider how infinately more complex body structures we humans have.

And I think the reasoning is fairly solid. I do now think that it is silly to imagine things like mitosis, DNA, human eyes could have appeared by random chance.

The book also goes into absolute lack of evidence in macro-evolution (aka missing links) and how the scientists were still unable to create life out of chemical reactions as they proclaim.

But I do not want to get into the macro evolution/ biogenesis theory stuff, but just would like a discussion on use of logic in the chair anology.

thanks.

[/ QUOTE ]

Now talk to a freind with some science background. ask him for some books on science. Read up some explanations of how what you see could have arisen from the laws of physics as we understand them. Then decide having seen both sides of the story. The reality of existance is far more intersting and beautiful than "I don't understand so it must be a God that that made it all". Read up some science stuff..it will really blow your mind:)

11-21-2005, 07:33 PM
[ QUOTE ]

Say you are walking down a field, and you see a wooden chair in middle of no where. Now you know someone must have made that chair, because chair's don't just "happen". You've never witnessed the builder of the chair working on the chair, but you could only assume that someone out there, made the chair and placed it there for whatever reason.


[/ QUOTE ]


Very unthinking. I would say that if you walked the tropical forests on a daily basis, you would find an enormous number of arrangements formed by fallen branches and trunks that look like chairs of all types.

In the evolutionary process there may be millions and millions of changes all in different directions and all random, only one of them may provide a better fit or a survival advantage.

Much more likely than (Un)intelligent Design fantasy (please lets not call it a scientific theory, it is nothing of the sort). Again, like all of those peddlers of beliefs, they will take examples that don't even serve their purposes. The eye is a case in point and an excellent example of bad design. It is far from optimal, has structural faults that are related to its evolutionary origin. Please do a search on the net. It is very basic scientific information. Just ignore the peddlers of ignorance (usually anything to do with theism).

11-21-2005, 07:34 PM
[ QUOTE ]
So my close christian friend lends me a book, because I've always been bugging her and questioning christianity, and she told me it'd answer most of my questions.

So I start reading the book and the very first chapter talks about this.

Say you are walking down a field, and you see a wooden chair in middle of no where. Now you know someone must have made that chair, because chair's don't just "happen". You've never witnessed the builder of the chair working on the chair, but you could only assume that someone out there, made the chair and placed it there for whatever reason.

Now if something as simple as a wooden chair can't just "happen" the book argues that something as complicated as humans, cell structures, plants, animals, countless laws of physics that govern the universe could not have just "happened". Somebody must have created us. If you think a wooden chair could not exist with out a builder, consider how infinately more complex body structures we humans have.

And I think the reasoning is fairly solid. I do now think that it is silly to imagine things like mitosis, DNA, human eyes could have appeared by random chance.

The book also goes into absolute lack of evidence in macro-evolution (aka missing links) and how the scientists were still unable to create life out of chemical reactions as they proclaim.

But I do not want to get into the macro evolution/ biogenesis theory stuff, but just would like a discussion on use of logic in the chair anology.

thanks.

[/ QUOTE ]

Buddy - you really need to read <u>The Blind Watchmaker</u> by Richard Dawkins. He has done enough to refute this simplistic argument.

Also, why do you say a chair is more "simple" than the biochemical reactions of life. (For instance, unless you switch to an anaerobic metabolism (like when you run and your muscles don't get enough oxygen), your body's chemistry is the same as buring wood.) What the Christian author did was frame the issue in manner that seems convincing, but the way he framed the issue was incorrect.

A chair is more complex that biochemical reactions when you look at the issue in this manner.

Biochemical reactions, all of them without question, simply follow the laws of thermodynamics. In other words the molecules behave the way they are supposed to behave (i.e., reactions that increase the amount entropy in the Universe). The chair on the other hand, was created by matter being forced to something it would not do on its own. Trees had to be cut, shaped and sanded.

The aetheist version on the origins of life is simply that "life" is the result of a mixture on chemicals (namely aromatic-carbon rings with nitrogen) behaving in full accord with the laws of thermodynamics.

Now, are you thinking that some one had to create the laws of thermodynamics? That to me is a better argument that a stinking chair? But still wrong, IMO. If we take for a <u>given</u> that the Universe exists, then everything in it has to behave in a predictable and certain fashion (i.e., the laws of thermodynamics). If it didn't, then you couldn't get drive to work because it would be a crap shoot as to whether or not gasoline felt like cumbusting with oxygen on that particular day. So if we exist, then there has to be a set of underlying rules. The two are inseparable, so why does there have to be a god because we have both.

Incidentally, it seems to me that "miracles" are supposed instances where the laws of thermodynamics or physics were supposedly violated in the presence of humans. So if a miracle (or violation of the laws of physics or thermodynamics) is proof of god, then how can nature acting as it should also be proof of god.

RJT
11-21-2005, 07:42 PM
Morph,

Sorry to interrupt. I never saw the underline instant UBB code before you just used it. I have needed it on several occasions. Is it new to the list of codes here? If it has always been there, then I need new glasses.

RJT

11-21-2005, 07:52 PM
[ QUOTE ]
Morph,

Sorry to interrupt. I never saw the underline instant UBB code before you just used it. I have needed it on several occasions. Is it new to the list of codes here? If it has always been there, then I need new glasses.

RJT

[/ QUOTE ]

It's in the Instant UBB code, at least on mine.

11-21-2005, 07:55 PM
[ QUOTE ]
So my close christian friend lends me a book, because I've always been bugging her and questioning christianity, and she told me it'd answer most of my questions.

[/ QUOTE ]

Here is another book you should read in order to get a balanced amount of input from opposing views, so that you may be more objective, and less biased:

[b]The Blind Watchmaker[b]: Why the Evidence of Evolution Reveals a Universe Without Design (http://www.amazon.com/gp/product/0393315703/103-0736652-2175043?v=glance&amp;n=283155&amp;n=507846&amp;s=books&amp;v=glanc e)

maurile
11-21-2005, 07:59 PM
[ QUOTE ]
The Blind Watchmaker: Why the Evidence of Evolution Reveals a Universe Without Design (http://www.amazon.com/gp/product/0393315703/103-0736652-2175043?v=glance&amp;n=283155&amp;n=507846&amp;s=books&amp;v=glanc e)

[/ QUOTE ]
The whole book is available online for free: LINK (http://www.evolutionary.tripod.com/dawkins_blindwatchmaker_1996_full.pdf).

11-21-2005, 08:03 PM
OT- Back several years, one of my teachers put up the arguement "do you think our face was arranged this way by chance? that our eyes are where they are by chance? there has to be a creator..."
I raised my hand and said "Well wouldn't you be saying the same thing if we had one eye on our backs?"

Shut him up for a week or so.

11-21-2005, 08:04 PM
[ QUOTE ]
[ QUOTE ]
The Blind Watchmaker: Why the Evidence of Evolution Reveals a Universe Without Design (http://www.amazon.com/gp/product/0393315703/103-0736652-2175043?v=glance&amp;n=283155&amp;n=507846&amp;s=books&amp;v=glanc e)

[/ QUOTE ]
The whole book is available online for free: LINK (http://www.evolutionary.tripod.com/dawkins_blindwatchmaker_1996_full.pdf).

[/ QUOTE ]

Awesome! Thanks for the link. I'd buy the book, myself (if I didn't already own it)... I still like to have the paper. But, for the e-reading-inclined, this PDF is awesome! Kudos.

RJT
11-21-2005, 08:13 PM
Kip,

You do realize that morphball already recommended the book. But, glad you did too as maurile provided a link to it free.

Thanks all, been looking for some Dawkins to read.

RJT

RJT
11-21-2005, 08:15 PM
[ QUOTE ]
[ QUOTE ]
Morph,

Sorry to interrupt. I never saw the underline instant UBB code before you just used it. I have needed it on several occasions. Is it new to the list of codes here? If it has always been there, then I need new glasses.

RJT

[/ QUOTE ]

It's in the Instant UBB code, at least on mine.

[/ QUOTE ]

Yeah, I see it now. Never saw it before and I looked before. Thanks.

chezlaw
11-21-2005, 08:17 PM
[ QUOTE ]
Kip,

You do realize that morphball already recommended the book. But, glad you did too as maurile provided a link to it free.

Thanks all, been looking for some Dawkins to read.

RJT

[/ QUOTE ]
The Ancestor's Tale is a cracking read. Top marks for the selfish gene as well. Plus the often forgotten Blind Watchmaker.

chez

11-21-2005, 08:25 PM
[ QUOTE ]
Kip,

You do realize that morphball already recommended the book. But, glad you did too as maurile provided a link to it free.

Thanks all, been looking for some Dawkins to read.

RJT

[/ QUOTE ]

Yeah, I saw that after I responded. But, now the OP has two links. One to buy the book, and one to read it for free! Hopefully he'll read it to get a more balanced view of Evolution.

I'm reading "The Selfish Gene" right now. And I also have "Unweaving the Rainbow" on my shelf to read.

Here's a quote from the back of the book:

[ QUOTE ]
Did Newton "unweave the rainbow" by reducing it to its prismatic colors, as Keats contended? Did he, in other words, diminish beauty? Far from it, says acclaimed scientist Richard Dawkins: Newton's unweaving is the key to much of modern astronomy and to the breathtaking poetry of modern cosmology. Mysteries don't lose their poetry because they are solved: the solution often is more beautiful than the puzzle, uncovering deeper mysteries.

[/ QUOTE ]

Allinlife
11-21-2005, 08:44 PM
cool thanks for the pdf. I'll read up on that

Bataglin
11-21-2005, 09:04 PM
It's probably a mutation /images/graemlins/grin.gif

RJT
11-21-2005, 09:39 PM
Ok guys, I need a quick answer.

Let me preface this by telling you that I like Dawkins. He seems a charming chap. And I will read him when I have more time. (Doing Soren K presently.) But I read the preface to <u>The Watchmaker</u> and the first sentence was this:


[ QUOTE ]
This book is written in the conviction that our own existence once presented the greatest of all mysteries, but that it is a mystery no longer because it is solved.

[/ QUOTE ]

So I skimmed ahead and he talks of the absurdity (my word not his) of going with a Creator. Then what he says about the origin of life is SPG spontaneous generation probability and leaves it at that (from I found with my brief skimming the book) . Tell me he has more than that for the origin life. Otherwise why the quoted first sentence?


RJT

hmkpoker
11-21-2005, 09:41 PM
[ QUOTE ]
OT- Back several years, one of my teachers put up the arguement "do you think our face was arranged this way by chance? that our eyes are where they are by chance? there has to be a creator..."
I raised my hand and said "Well wouldn't you be saying the same thing if we had one eye on our backs?"

Shut him up for a week or so.

[/ QUOTE ]

Ooooooh, that's a good one. I like it! /images/graemlins/smile.gif

Ten points and a gold star

gumpzilla
11-21-2005, 09:58 PM
I think one substantial flaw in the design analogy is this. Part of the reason that you suspect the chair in the middle of the glade is designed is because it is an instance of an artifact that IS designed and manufactured by humans. So in this case, it's an excellent assumption to believe that it was designed and put there, because you have lots of previous evidence about chairs being manufactured. But you don't have any kind of evidence like that for mitochondria, Gila monsters or the eye. So assuming that they must have been designed isn't as safe an assumption.

EDIT: Now that I've actually read the rest of the thread, I see maurile made this point as well.

maurile
11-21-2005, 10:24 PM
[ QUOTE ]
So I skimmed ahead and he talks of the absurdity (my word not his) of going with a Creator.

[/ QUOTE ]
He doesn't write about the absurdity of "going with" a Creator. He refutes a particular argument for a Creator -- namely, the watchmaker argument. He refutes it because it's a fallacious argument.

[ QUOTE ]
Then what he says about the origin of life is SPG spontaneous generation probability and leaves it at that (from I found with my brief skimming the book).

[/ QUOTE ]
I have no idea what part of the book you're referring to. Dawkins never wrote anything positive about "spontaneous generation." He has given a couple of plausible conjectures -- one in The Selfish Gene, and another in The Blind Watchmaker -- for how things may have gotten started. But he cautions that they are just conjectures. And spontaneous generation is not among them.

[ QUOTE ]
Tell me he has more than that for the origin life. Otherwise why the quoted first sentence?

[/ QUOTE ]
He's not referring to the origin of life (i.e., replicators) in the first quoted sentence. He's referring to the existence of complex organisms and biological features.

We don't know how the first replicators came into existence (though we can speculate about a few of the possibilities). But once those replicators did come into existence, their evolution into complex biological features such as the human eye is well understood. It is "a mystery no longer because it is solved."

RJT
11-21-2005, 11:04 PM
maurile,

I should have just quoted him directly. I was going to but couldn’t find what I read (plus one can‘t cut and paste from Adobe - I couldn‘t anyway). I kind of misrepresented what he said. But my point is still the same.

Page 141:

[ QUOTE ]
To explain the origin of DNA/protein machine by invoking a supernatural Designer is to explain precisely nothing, for it leaves unexplained the origin of the Designer. You have to say something like ‘God was always there’, and if you allow yourself that kind of lazy way out, you might just say ‘DNA was always there’, or ‘Life was always there’, and be done with it.

[/ QUOTE ]

He doesn’t refute The Blind Watchmaker at all. He simply dismisses it as not necessary. (page 147 and on). Then he goes on to say “We are here concerned with the kind of solution that must be found…”, (to figure out the origin of life).

This is why I think we will never understand each other (atheist and belivers). You guys seem to think he is saying something. I mean he is and he is interesting and the science must be fascinating. But, he doesn’t say anything of the origin of life. There is nothing in his methodology that leads one to a conclusion that God won’t be at the end of the tunnel. Perhaps not the God we all talk about. But, in simply dismissing the Watchmaker he says nothing. And in simply talking about how to find the solution he says nothing again.

I guess what I am really trying to say is that we believers and atheists (who read stuff like this) are really talking about the same thing. We just speak different languages is all.

RJT

maurile
11-21-2005, 11:46 PM
[ QUOTE ]
He doesn’t refute The Blind Watchmaker at all.

[/ QUOTE ]
Of course not. He wrote it. He does, however, refute the watchmaker argument for design. It's the point of his whole book, and he is strikingly successful.

[ QUOTE ]
He simply dismisses it as not necessary.

[/ QUOTE ]
He shows with convincing argument that it is not necessary. That's a lot different from "simply dismissing it."

[ QUOTE ]
But, he doesn’t say anything of the origin of life. There is nothing in his methodology that leads one to a conclusion that God won’t be at the end of the tunnel.

[/ QUOTE ]
No. Why would he?

The Blind Watchmaker isn't an argument for atheism. It's a refutation of a specific argument for theism.

If you say it must be raining in Cleveland because the Packers just scored a touchdown, I will refute your argument by showing that just because the Packers scored a touchdown doesn't mean that it must be raining in Cleveland. In doing so, however, I would not be arguing that it's not raining in Cleveland. It may be, for all I know. But I've still refuted your reason for thinking that it is.

Similarly, Dawkins persuasively refutes the notion that just because a human eye looks designed, God must exist. Maybe He does, maybe He doesn't. Maybe it's raining in Cleveland. But the watchmaker argument is refuted.

RJT
11-22-2005, 12:01 AM
Maurile,

[ QUOTE ]
He shows with convincing argument that it is not necessary. That's a lot different from "simply dismissing it."

[/ QUOTE ]

But he does not refute it is my point You had said:

[ QUOTE ]
He refutes it because it's a fallacious argument.

[/ QUOTE ]

I meant to type “the Watchmaker” (not include the word Blind) which I think is the analogy to a creator then. Right? Again, I read this briefly.

RJT

RJT
11-22-2005, 12:18 AM
[ QUOTE ]
But, he doesn’t say anything of the origin of life. There is nothing in his methodology that leads one to a conclusion that God won’t be at the end of the tunnel.

[/ QUOTE ]

[ QUOTE ]
No. Why would he?

The Blind Watchmaker isn't an argument for atheism. It's a refutation of a specific argument for theism.

[/ QUOTE ]

I think you misread what I wrote. Without the double negatives, I’ll re-write. His methodology does not eliminate a god at the end of the tunnel. That a god is/or is not the source of life. Or do I need to continue reading? What I gathered was on page 164 and the probabilities of life is what he talks about.

maurile
11-22-2005, 12:25 AM
[ QUOTE ]
His methodology does not eliminate a god at the end of the tunnel.

[/ QUOTE ]
Right. He doesn't refute the idea that Bush's tax cuts were wise, either. That's not the point.

As I said in my last post, he refutes the watchmaker argument -- i.e., the idea that since the eye looks designed, therefore God must exist.

RJT
11-22-2005, 12:32 AM
[ QUOTE ]
[ QUOTE ]
His methodology does not eliminate a god at the end of the tunnel.

[/ QUOTE ]
Right. He doesn't refute the idea that Bush's tax cuts were wise, either. That's not the point.

As I said in my last post, he refutes the watchmaker argument -- i.e., the idea that since the eye looks designed, therefore God must exist.

[/ QUOTE ]

Maurile,

Yeah, that‘s fine.

I really am concerned (now that we are on the same page with the other stuff) with his opening statement.

[ QUOTE ]
This book is written in the conviction that our own existence once presented the greatest of all mysteries, but that it is a mystery no longer because it is solved.

[/ QUOTE ]

I don’t see him fulfilling the promise his all.

RJT

maurile
11-22-2005, 12:37 AM
[ QUOTE ]
[ QUOTE ]
[ QUOTE ]
His methodology does not eliminate a god at the end of the tunnel.

[/ QUOTE ]
Right. He doesn't refute the idea that Bush's tax cuts were wise, either. That's not the point.

As I said in my last post, he refutes the watchmaker argument -- i.e., the idea that since the eye looks designed, therefore God must exist.

[/ QUOTE ]

Maurile,

Yeah, that‘s fine.

I really am concerned (now that we are on the same page with the other stuff) with his opening statement.

[ QUOTE ]
This book is written in the conviction that our own existence once presented the greatest of all mysteries, but that it is a mystery no longer because it is solved.

[/ QUOTE ]

I don’t see him fulfilling the promise his all.

RJT

[/ QUOTE ]
He does fulfill it. You're just reading it wrong. You seem to be reading it as if he's promising to prove that God doesn't exist. That's not what he's promising, and it's not what he does. What he does is show how evolution can ape conscious design by increasing complexity over succeeding generations. It can make stuff like mammalian eyes that appear to have been designed. In doing so, he refutes the idea that just because the eye looks designed, God must exist.

Lestat
11-22-2005, 12:42 AM
[ QUOTE ]
[ QUOTE ]
Quote:
This book is written in the conviction that our own existence once presented the greatest of all mysteries, but that it is a mystery no longer because it is solved.

[/ QUOTE ]



I don’t see him fulfilling the promise his all.

RJT

[/ QUOTE ]

He's referring to evolution and fulfills the promise by writing the rest of the book.

RJT
11-22-2005, 12:47 AM
Right, I get all that. I got that with Darwin and the Leakeys. He solves no mysteries is all I am saying. He tells a good detailed version of evolution is all and ways to solve things going back further in time up to the point of where our knowledge now begins.

RJT
11-22-2005, 12:48 AM
[ QUOTE ]
[ QUOTE ]
[ QUOTE ]
Quote:
This book is written in the conviction that our own existence once presented the greatest of all mysteries, but that it is a mystery no longer because it is solved.

[/ QUOTE ]



I don’t see him fulfilling the promise his all.

RJT

[/ QUOTE ]

He's referring to evolution and fulfills the promise by writing the rest of the book.

[/ QUOTE ]

That makes sense, Stat.

maurile
11-22-2005, 12:56 AM
[ QUOTE ]
He solves no mysteries is all I am saying.

[/ QUOTE ]
He solves the mystery William Paley used as the basis for his Watchmaker argument. (Actually, Dawkins doesn't solve it. Darwin and Wallace solved it. Dawkins describes the solution.)

RJT
11-22-2005, 01:02 AM
[ QUOTE ]
[ QUOTE ]
[ QUOTE ]
Quote:
This book is written in the conviction that our own existence once presented the greatest of all mysteries, but that it is a mystery no longer because it is solved.

[/ QUOTE ]



I don’t see him fulfilling the promise his all.

RJT

[/ QUOTE ]

He's referring to evolution and fulfills the promise by writing the rest of the book.

[/ QUOTE ]

Yet, if he is writing to the believer of evolution; I don’t see his need (in the preface) to make his own case as an apologist for evolution. If he is writing also to the believer in a creator God (not necessarily one who buys into ID) he changes nothing. Those believers who can live in both worlds (creator God and evolution) have a greater understanding of evolution. Those who can’t, I am not sure that he says anything since he talks nothing of actual origin of life.

RJT
11-22-2005, 01:04 AM
[ QUOTE ]
[ QUOTE ]
He solves no mysteries is all I am saying.

[/ QUOTE ]
He solves the mystery William Paley used as the basis for his Watchmaker argument. (Actually, Dawkins doesn't solve it. Darwin and Wallace solved it. Dawkins describes the solution.)

[/ QUOTE ]

So this text needs a context to understand his reason for writing it? His preface is misleading (to me) is basically what I am getting at.

maurile
11-22-2005, 01:14 AM
RJT, you should just read the book. I don't see how it's possible for you to be so misled by it, and to so misunderstand his point, if you read it. It's written very clearly.

Lestat
11-22-2005, 01:19 AM
[ QUOTE ]
[ QUOTE ]
[ QUOTE ]
[ QUOTE ]
Quote:
This book is written in the conviction that our own existence once presented the greatest of all mysteries, but that it is a mystery no longer because it is solved.

[/ QUOTE ]



I don’t see him fulfilling the promise his all.

RJT

[/ QUOTE ]

He's referring to evolution and fulfills the promise by writing the rest of the book.

[/ QUOTE ]

Yet, if he is writing to the believer of evolution; I don’t see his need (in the preface) to make his own case as an apologist for evolution. If he is writing also to the believer in a creator God (not necessarily one who buys into ID) he changes nothing. Those believers who can live in both worlds (creator God and evolution) have a greater understanding of evolution. Those who can’t, I am not sure that he says anything since he talks nothing of actual origin of life.

[/ QUOTE ]

He is not putting the reader into any type of class. He doesn't care if the reader is an atheist, a theist, or a numerologist. He simply presents the case for how Darwin's Theory of Evolution can beautifully explain what was once unexplainable (except for an intelligent designer).

You're going wrong in thinking he has some hidden agenda to promote atheism. He does not.

RJT
11-22-2005, 01:22 AM
[ QUOTE ]
RJT, you should just read the book. I don't see how it's possible for you to be so misled by it, and to so misunderstand his point, if you read it. It's written very clearly.

[/ QUOTE ]


Maurile,

I am not mislead by the book, only the preface.

Someday, I might read the book. I really don’t have the time to read about evolution in detail. It is interesting, but not something I care to learn the details about. It does not conflict with my beliefs so I have no “problem” with it.

Let me put it this way. We are reading Soren K. in the book club here on the forum as you know. It is good to know that Soren is really basically talking to Hegel. But one does not need to know that to follow his book. (If one can follow it - lol.) I think without this knowledge that you and Stat posted that his preface is not clear. If you have time, re-read the preface from my point of view and see if I missed something/misread it or if I am correct.

RJT

maurile
11-22-2005, 01:25 AM
[ QUOTE ]
I think without this knowledge that you and Stat posted that his preface is not clear.

[/ QUOTE ]
Everything we've posted is right from the preface.

RJT
11-22-2005, 01:26 AM
I am not saying he is trying to promote atheism. I am saying evolution is not mystery to me. Who is he trying to convince? He solves no other mystery is what I am saying. Re-read his preface is all I ask.

RJT
11-22-2005, 01:27 AM
[ QUOTE ]
[ QUOTE ]
I think without this knowledge that you and Stat posted that his preface is not clear.

[/ QUOTE ]
Everything we've posted is right from the preface.

[/ QUOTE ]

Ok I will re-read it then.

maurile
11-22-2005, 01:28 AM
[ QUOTE ]
I am not saying he is trying to promote atheism. I am saying evolution is not mystery to me. Who is he trying to convince? He solves no other mystery is what I am saying. Re-read his preface is all I ask.

[/ QUOTE ]
The preface describes William Paley's argument from design (the "watchmaker argument") and then explains why, in light of Darwin's and Wallace's discoveries, it is no longer compelling.

I don't see how that's misleading.

RJT
11-22-2005, 01:40 AM
Stat and maurile,

Ok, I got all that you two said. My only contention comes down now to his first sentence. Really his use of the word “mystery”. Did he solve any mystery for you guys? I read too much into his use of the word mystery is what happened. This threw me off as I then anticipated more than a book only about evolution.

I guess I am one of those who was “unaware that there was a problem in the first place”.

Thanks for your patience.

RJT

Aytumious
11-22-2005, 01:43 AM
[ QUOTE ]
Stat and maurile,

Ok, I got all that you two said. My only contention comes down now to his first sentence. Really his use of the word “mystery”. Did he solve any mystery for you guys? I read too much into his use of the word mystery is what happened. This threw me off as I then anticipated more than a book only about evolution.

I guess I am one of those who was “unaware that there was a problem in the first place”.

Thanks for your patience.

RJT

[/ QUOTE ]

RJT, the irony here is that you are saying you don't see the need for the book when this thread itself is a perfect example of why the book was written.

maurile
11-22-2005, 01:49 AM
[ QUOTE ]
Did he solve any mystery for you guys?

[/ QUOTE ]
http://forumimages.footballguys.com/style_emoticons/default/wallbash.gif

Lestat
11-22-2005, 02:03 AM
[ QUOTE ]

http://forumimages.footballguys.com/style_emoticons/default/wallbash.gif

[/ QUOTE ]

I LOVE IT!!!

RJT
11-22-2005, 02:11 AM
[ QUOTE ]
[ QUOTE ]
Did he solve any mystery for you guys?

[/ QUOTE ]
http://forumimages.footballguys.com/style_emoticons/default/wallbash.gif

[/ QUOTE ]


LMAO. I have more interest in knowing how you did that head banging thing, then details of evolution. Good work.

[ QUOTE ]
The preface describes William Paley's argument from design (the "watchmaker argument") and then explains why, in light of Darwin's and Wallace's discoveries, it is no longer compelling.

[/ QUOTE ]

What, did you guys need convincing that Paley’s argument is no longer compelling? Of course not. I didn’t. Like I said I didn’t even know there was a problem to begin with. What am I missing? Who is Dawkins trying to convince?

What Arty? I buy into the evolution thing. You guys study it and tell me all the details I need to know to live my life. Seriously, I don’t get your point - unless it is not directed at me, rather to the OP?

RJT

Lestat
11-22-2005, 02:50 AM
RJT-

I think you may be misunderstaning what Dawkins meant by mystery. He did not mean the mystery of God, the universe, or what it all means.

Before Darwin, there was a lot that was very much a mystery!

So if you buy into the whole evolution thing, but are looking for him to solve other mysteries such as the creation of the universe, this isn't what the book is about.

If I've understood you correctly, evolution has little to do with your theistic beliefs anyway. As I remember it, the whole premise with you eminates from questions such as how the universe started or whether it was always there. This book will do nothing to dispell those beliefs. It is a very interesting read nonetheless.

RJT
11-22-2005, 03:17 AM
Thanks Stat,

I got it now. I read it very quickly - the preface and chapter 6. I was merely thrown off when Dawkins started his preface with so much of an argument for evolution. I therefore thought he was directing it basically to those who don’t buy it. He isn't going to get anywhere with them anyway. So this, too, is what I mean in that he fails. He is preaching to the choir, really. He is really addressing evolutionists.

As for myself, I didn’t expect answers to the origin of life, per se. That I know “ain’t gonna” happen. I was anticipating some clever ideas is all. He doesn’t address things of that nature.

RJT

chezlaw
11-22-2005, 04:51 AM
[ QUOTE ]
Thanks Stat,

I got it now. I read it very quickly - the preface and chapter 6. I was merely thrown off when Dawkins started his preface with so much of an argument for evolution. I therefore thought he was directing it basically to those who don’t buy it. He isn't going to get anywhere with them anyway. So this, too, is what I mean in that he fails. He is preaching to the choir, really. He is really addressing evolutionists.

As for myself, I didn’t expect answers to the origin of life, per se. That I know “ain’t gonna” happen. I was anticipating some clever ideas is all. He doesn’t address things of that nature.

RJT

[/ QUOTE ]
Maybe its due to books like this that you don't percieve evolution of complex life as a mystery. I read it 20 or so years ago and it was a revelation (not that I was a theist before).

Many people still seem very confused and although it may not persuade a theist that evolution is the answer it should go a long way to persuading anyone who is honest that the argument from design is extremely weak to the point of being irrelevent.

I have said before that Dawkins goes over the top on the athiestic promotion and I would be suprised if that didn't put some people off from reading his work.

chez

11-22-2005, 08:20 AM
RJT - I see a lot of disagreements to your post about Dawkins sentence saying the mystery of our existence has been solved. I'll lend you some support - I think your instincts are good in questioning this kind of a statement. Personally I think there are plenty of mysteries that remain pertaining to the origin of life and to our existence.
I've been through Dawkin's book "The Blind Watchmaker " twice about 5 years apart. On the positive side I think it adresses some of the central questions about evolution that need to be adressed. It does a good job in some places of making points. But overall I think Dawkin's case is weak in proving the mechanisms behind the so called Neo-Darwinian model of evolution. There are so many intracasies in this discussion that it is impossible to deal with them in short posts. But I cast my vote that the idea of intelligent design is not a trivial thing to dismiss. Whether the original poster's chair represents certain structures in the body, certain forms of life or the Universe itself - plenty of scientists that I have talked to at Universities would say its a reasonable discussion. (I've taught college physics but not biology).

Lestat
11-22-2005, 09:52 AM
A physics teacher who isn't willing to dismiss intelligent design?

You are truly in a minority.

NotReady
11-22-2005, 01:02 PM
But I do not want to get into the macro evolution/ biogenesis theory stuff, but just would like a discussion on use of logic in the chair anology.

The argument you are considering is the theistic proof usually referred to as the teleological proof. Paley is famous for stating it, though in a sense it is also contained in Scripture, though not offered there as a formal philosopical proof ("The heavens are telling of the glory of God" and other passages.) It's a very strong argument because it appeals to our intuition - we know deep down that order doesn't come from chaos and order as great and complex as displayed throughout the universe implies a designer beyond our ability to comprehend.

There are 5 proofs considered to be classic, and many variations. The problem is none of them are perfect - none achieve objective, demonstrable certainty. The reason for this is primarily that man is finite and so must start his reasoning process from certain assumptions which can't be proved. So the logic may be perfect but the argument cannot be made certain because of the need for assumptions. This same problem afflicts atheists as well no matter how strongly people like Dawkins assert they have solved the mystery of life. Every time they think they have they just uncover many new mysteries exposed by their "answer".

Paul stated that he did not come with clever words of wisdom and he indicated that Christianity is not based on philosophical reasoning. Our faith rests on the Word of God, not on the flawed and sinful imaginations of man. That doesn't mean that we are irrational or that we have no reasons for believing. But we can't present absolute proof, anymore than the atheist can do so concerning his theories.

We walk by faith and admit it. Atheists walk by faith and don't realize it.

NotReady
11-22-2005, 01:04 PM
[ QUOTE ]

You're going wrong in thinking he has some hidden agenda to promote atheism. He does not.


[/ QUOTE ]

Correct. He doesn't hide it at all.


http://www.atheistalliance.org/library/news_021703.html

The Richard Dawkins Award will be given every year to honor an outstanding Atheist whose contributions raise public awareness of the nontheist life stance; who through writings, media, the arts, film, and/or the stage advocates increased scientific knowledge; who through work or by example teaches acceptance of the nontheist philosophy; and whose public posture mirrors the uncompromising nontheist life stance of Dr. Richard Dawkins.

jthegreat
11-22-2005, 01:56 PM
[ QUOTE ]
It's a very strong argument because it appeals to our intuition

[/ QUOTE ]

Unfortunately it's a false analogy. First it uses nature as the example of chaos, then claims it's perfectly ordered.

Weak.

11-22-2005, 03:30 PM
[ QUOTE ]
The reason for this is primarily that man is finite and so must start his reasoning process from certain assumptions which can't be proved.

[/ QUOTE ]

Some people just have more assumptions than others. One of my assumptions, is that it's better to have fewer assumptions.

Occam's Razor: "One should not increase, beyond what is necessary, the number of entities required to explain anything." (http://en.wikipedia.org/wiki/Occam's_Razor)

hmkpoker
11-22-2005, 04:11 PM
[ QUOTE ]
Atheists walk by faith and don't realize it.

[/ QUOTE ]

If you have the time and incentive, please elaborate on this.

11-22-2005, 04:32 PM
[ QUOTE ]
[ QUOTE ]
Atheists walk by faith and don't realize it.

[/ QUOTE ]

If you have the time and incentive, please elaborate on this.

[/ QUOTE ]

He's equivocating the word "faith". We've been through all this before. He's saying that induction is "faith" (or at least thinking that induction is valid/reasonable is).

11-22-2005, 04:36 PM
[ QUOTE ]
I was merely thrown off when Dawkins started his preface with so much of an argument for evolution. I therefore thought he was directing it basically to those who don’t buy it. He isn't going to get anywhere with them anyway. So this, too, is what I mean in that he fails. He is preaching to the choir, really. He is really addressing evolutionists.

[/ QUOTE ]

He's addressing those that either don't know how complex our biological systems are, or those that think this complexity necessitates design.

Here is the preface (for those that want to reference it without downloading the entire PDF):

[ QUOTE ]
This book is written in the conviction that our own existence once
presented the greatest of all mysteries, but that it is a mystery no
longer because it is solved. Darwin and Wallace solved it, though we
shall continue to add footnotes to their solution for a while yet. I wrote
the book because I was surprised that so many people seemed not only
unaware of the elegant and beautiful solution to this deepest of
problems but, incredibly, in many cases actually unaware that there
was a problem in the first place!
The problem is that of complex design. The computer on which I am
writing these words has an information storage capacity of about 64
kilobytes (one byte is used to hold each character of text). The
computer was consciously designed and deliberately manufactured.
The brain with which you are understanding my words is an array of
some ten million kiloneurones. Many of these billions of nerve cells
have each more than a thousand 'electric wires' connecting them to
other neurones. Moreover, at the molecular genetic level, every single
one of more than a trillion cells in the body contains about a thousand
times as much precisely-coded digital information as my entire
computer. The complexity of living organisms is matched by the
elegant efficiency of their apparent design. If anyone doesn't agree that
this amount of complex design cries out for an explanation, I give up.
No, on second thoughts I don't give up, because one of my aims in the
book is to convey something of the sheer wonder of biological
complexity to those whose eyes have not been opened to it. But having
built up the mystery, my other main aim is to remove it again by
explaining the solution.

[/ QUOTE ]

hmkpoker
11-22-2005, 04:49 PM
[ QUOTE ]
[ QUOTE ]
[ QUOTE ]
Atheists walk by faith and don't realize it.

[/ QUOTE ]

If you have the time and incentive, please elaborate on this.

[/ QUOTE ]

He's equivocating the word "faith". We've been through all this before. He's saying that induction is "faith" (or at least thinking that induction is valid/reasonable is).

[/ QUOTE ]

You mean like how I don't really think about how the engine/keylock mechanism works when I start my car, I just kinda know that it'll start when my key goes in and turns?

Bah, I want to hear this from the horse's mouth /images/graemlins/smile.gif

jthegreat
11-22-2005, 05:03 PM
[ QUOTE ]
He's equivocating the word "faith". We've been through all this before. He's saying that induction is "faith" (or at least thinking that induction is valid/reasonable is).

[/ QUOTE ]

More specifically, he is stating that the idea of an orderly universe, which is required for the validity of science, is based on an unprovable assumption and therefore must be taken on faith.

However, as I stated in the thread I started earlier today, that the universe is currently ordered is axiomatic.

college kid
11-22-2005, 05:32 PM
Sorry, but no. Your argument is flawed in numerous ways. First of all, as somebody pointed out, if God does exist, then who or what created God, ad infinitum. Secondly, you must take into consideration what exactly IS a woonden chair. What makes it a wooden chair. Must it be polished and be able to rock back and forth? What about a tree stump with a back? Your argument assumes that you KNOW what a wooden chair is and that you find one. But if you didn't know what a wooden chair was supposed to look like it would not seem out of the ordinary. Now suppose that the tree stump with a back on it, provides some extra use to the tree and over time, more trees appear that way. Suddenly the tree has evolved into a wooden chair.

The analogy is flawed because human aren't like your wooden chair in that there was no preset way for us to be or for what we should look like. Now ask yourself how probable it is that you find something that kind of might resemble a piece of furniture in the middle of nowhere. More probable??? You say it's silly to think that we appeared by random chance? Why? The universe is so vast and old that even an improbable occurance like life CAN happen. And once it does, it can morph and evolve any number of ways. If you take human beings as we are now and say that it's improbable, you are right. But if you think about the idea that we are just one of many possible directions early life could have evolved into, the probability of our occurance or something else is HIGHLY likely.

If you pick a number 1-20, chances are that it won't be 16. But you ARE picking a number and the fact that it happened to be 16 doesn't make 16 significant.

I don't know anything about biology or life or really a whole lot about religion for that matter, but I hope you see the flaw in your argument. You are assuming that your wooden chair MUST have come to be a certain way because that's how it is now. But if you backtrack and think that the outcome was only one of many possible outcomes, you find that the occurance of a wooden chair is more likely. Simple Bayes.

Of course I am continually criticised on this forum for my stupidity, so if somebody would care to pick me apart and call me an idiot, maybe you can just forget my post!

11-22-2005, 05:45 PM
[ QUOTE ]
[ QUOTE ]
He's equivocating the word "faith". We've been through all this before. He's saying that induction is "faith" (or at least thinking that induction is valid/reasonable is).

[/ QUOTE ]

More specifically, he is stating that the idea of an orderly universe, which is required for the validity of science, is based on an unprovable assumption and therefore must be taken on faith.

However, as I stated in the thread I started earlier today, that the universe is currently ordered is axiomatic.

[/ QUOTE ]

Yeah... and we know the universe is ordered by induction. We examine it. Since every oberservation has shown it to be ordered, we induce that it is ordered. And the sun will come up(*) tomorrow. I know that by faith, of course. /images/graemlins/wink.gif

(*) Note: by "come up", I mean that the rotation of the earth will bring the sun into our line of sight, in a way that it appears to be "coming up". /images/graemlins/laugh.gif

jthegreat
11-22-2005, 07:23 PM
Induction doesn't provide certainty, though. Axioms do.

Scotch78
11-22-2005, 08:23 PM
This is called the argument by design, and I'm surprised that a gambler would be taken in by it. I have a standard six-sided die, what are the odds of me rolling a 1? Okay, I just rolled a one, now what are the odds of me having rolled a 1?

Scott

Jeff V
11-22-2005, 08:32 PM
[ QUOTE ]
If you pick a number 1-20, chances are that it won't be 16. But you ARE picking a number and the fact that it happened to be 16 doesn't make 16 significant.

[/ QUOTE ]

Not significant, but it establishes probability, which can help us decide how likely the outcome would be and how much we should be concerned about such an outcome.

Biochemist Michael Behe said the probabliity of linking 100 amino acids to create one protein molecule would be the same as a blindfolded man finding a marked grain of sand in the Sahara desert- not once but 3 different times.

11-22-2005, 11:16 PM
[ QUOTE ]
Induction doesn't provide certainty, though. Axioms do.

[/ QUOTE ]

Science doesn't provide certainty either. It provides cogency.

IbrakeFORrivers
11-23-2005, 02:35 AM
read St. Augustine... he goes into a lot of Plato-nian concepts... Read Timaeus by Plato... It's interesting stuff...

11-23-2005, 05:38 AM
[ QUOTE ]

Biochemist Michael Behe said the probabliity of linking 100 amino acids to create one protein molecule would be the same as a blindfolded man finding a marked grain of sand in the Sahara desert- not once but 3 different times.

[/ QUOTE ]

what is timetable for the blind man? his natural lifetime? a day? before he dies of thirst?
sorry, not at all relevant to the discussion, but part of me really wants that quantified. it seems too specific to simply be a hyperbolic statement on Behe's part.

11-23-2005, 06:14 AM
[ QUOTE ]
Biochemist Michael Behe said the probabliity of linking 100 amino acids to create one protein molecule would be the same as a blindfolded man finding a marked grain of sand in the Sahara desert- not once but 3 different times.

[/ QUOTE ]

LOL... What a statement. That will get him maybe to appear on Oprah or the pre-school lecture circuit, but it surely will not get him closer to the Nobel prize than the blind man to a specific grain of sand in the desert.

It is amazing that creationists let people loose with statements like that. I mean it is evident why they are a laughing stock.

Lets look at the statement. First, of course is the objection already raised. It doesn't quantify anything. How long has he got etc...

But the grossest error committed by the biochemist (? obviously with zero knowledge of statistics or science) is that he fails, by insidiously putting forward an erroneous view pushed by creationists. That somehow, specifics known linking of 100 amino acids are the ONLY way to create a protein molecule useful to life. as they are are something remarkable or peculiar and somehow they only one that could be related to the phenomena of life. Of course, a scientist make no such claim. We have far from exhausted investigating all possible proteins structures (if in themselves they are neccesary to life and there are NO other possibilities. Not known either). To put the analogy a bit more correctly, altough still lacking in quantification, assume that the sand is made up af yellow, black and red grains. Now the probability may be mooted a bit more accurately by saying it would be equivalent to the likehood of the blind man picking three yellow grains in a row if he kept picking grain all his life.

Anyway, a very interesting observation... at least as an indication of the level of intellectual prowesses by Mr Behe. LOL again.

college kid
11-23-2005, 07:36 AM
Again, I'm out of my element here. But it IS possible, and given enough time/trials it WILL happen. So just because it's unlikely to happen at all doesn't mean that once it has happened, it therefore must be something devine. We just put a bad beat on probabilistic natural occurance. What a suck out. NH.

jthegreat
11-23-2005, 09:48 AM
[ QUOTE ]
Science doesn't provide certainty either. It provides cogency.

[/ QUOTE ]

That's not the point. The point is that you can be certain that nature is ordered because it's axiomatic. If you "knew" through induction, you wouldn't be 100% certain.

11-23-2005, 11:42 AM
Calculating the odds of a single complex structure arising by the Neo-Darwinian model of evolution is not an idea that is automatically trumped by pointing out that there are many possible complex structures that could have evolved.
I like thinking of cars rather than chairs. I calculate the odds of a Corvette arising by the Neo-Darwinian model of evolution. Someone says - well a Porshe could have evolved or a Mazda or Lord help us a Yugo - even a plane - even a boat etc. That doesn't trump the argument. Its just one more thing to consider in the calculation. And it is considered by many people who advance the intelligent design arguments.
Some people here are crying for quantification and you are right. Youve got to look at the math and the specifics. Just as a rough example - The odds of picking one correct number in a lottery with 4 numbers is one in 10 to the 4th power - or 1 in 10,000. But if I tell you that you have 1,000 numbers that are winners your odds aren't that bad. But what if the lottery has 1 million numbers and you only have 1,000 chances (the corvette, the porshe, the boat and the plane etc...). So whether you have one chance or a thousand chances at a lottery with one million digits doesn't really change the argument of the likelyhood of winning.
So its important to look specifically and thoroghly at these kinds of questions. And thats where Micheal Behe's most famous book, "Darwin's Black Box", does a good job. The book is a tough read though because it gets into a lot of biology and biochemistry, but you can still follow a lot of his arguments.
Also - Behe careful - I see comments about Behe being a crazy Creationist and unqualified in his field. He is well qualified, and Behe is not a creationist. He has stated he prefers evolution over creation. There are really 3 camps out there now:
1. Evolutionists who hold pretty much to the Neo-Darwinian model of Evolution, allowing maybe a few tweeks to come along in the future.
2. Creationists who are motivated mainly to prove a set of religious beliefs.
3. Those who may agree with evolution as a general principle but are challenging the Neo-Darwinian model. (Also in this camp I think are people who more or less believe in creation but are not pushing a literal interpretation of Genesis. Perhaps that deserves a 4th camp for now putting them together is OK)
So Behe is in camp #3. And so am I. I believe there are serious difficulties with Neo-Darwinism as a mechanism for evolution. Part of the problem is that the evolutionists have been under fire from camp #2 for so long that they are tired of the debate. Camp #2 has been motivated by proving specific religious beliefs. But camp #2 has hit the evolutionists with some very legitimate points over the years. And when camp #3 uses some of the same points, they get labelled as being in camp #2 and the walls go up.
Maybe I will do a thread some day on the objections to the Neo-Darwinian model of evolution and some of the perspectives of camp #3.

noggindoc
11-23-2005, 11:57 AM
[ QUOTE ]
So my close christian friend lends me a book, because I've always been bugging her and questioning christianity, and she told me it'd answer most of my questions.

So I start reading the book and the very first chapter talks about this.

Say you are walking down a field, and you see a wooden chair in middle of no where. Now you know someone must have made that chair, because chair's don't just "happen". You've never witnessed the builder of the chair working on the chair, but you could only assume that someone out there, made the chair and placed it there for whatever reason.

Now if something as simple as a wooden chair can't just "happen" the book argues that something as complicated as humans, cell structures, plants, animals, countless laws of physics that govern the universe could not have just "happened". Somebody must have created us. If you think a wooden chair could not exist with out a builder, consider how infinately more complex body structures we humans have.

And I think the reasoning is fairly solid. I do now think that it is silly to imagine things like mitosis, DNA, human eyes could have appeared by random chance.

The book also goes into absolute lack of evidence in macro-evolution (aka missing links) and how the scientists were still unable to create life out of chemical reactions as they proclaim.

But I do not want to get into the macro evolution/ biogenesis theory stuff, but just would like a discussion on use of logic in the chair anology.

thanks.

[/ QUOTE ]

Geez man this argument is old and lame. Why can't random interactions over an infinite period of time produce what we perceive to be complexity? You've not heard the one where a monkey randomly banging on a typewriter over an infinite amount of time would eventually write all the books known to man?

jthegreat
11-23-2005, 12:13 PM
[ QUOTE ]
3. Those who may agree with evolution as a general principle but are challenging the Neo-Darwinian model. (Also in this camp I think are people who more or less believe in creation but are not pushing a literal interpretation of Genesis. Perhaps that deserves a 4th camp for now putting them together is OK)


[/ QUOTE ]

Behe is not simply challenging the model. He is presenting an alternative "hypothesis" that is *philosophical*, *not* scientific in nature. He is not simply saying "This model doesn't explain this well. Let's think of other biological means for this occurrence." He's saying "This model doesn't explain this well. Therefore the model is totally wrong and let's use an untestable explanation instead."

It's a bunch of crap that any intelligent person should be able to see through. It's equivalent to saying "Since Newtonian physics doesn't completely accurately describe physical motion, let's just say that ghosts move things around."

11-23-2005, 12:15 PM
On the one hand one person can say life is infinitely complex so it could not have evolved. On the other hand another pserson can say you have an infinite number of tries to create life so it will eventually happen. Neither side wins this debate. Life is very complex, but not infinitely complex. And there is not an infinite amount of time. Spontaneous generation of life somewhere in the Universe is one thing, but basically evolution is saying we went from a monkey type brain to a human brain in about 2 million years. The brain is really complex on the one hand - on the other hand 2 million years is a long time. So you've got to get into the specifics of the problem. Brains are not the only thing to look at. There are pretty good time estimates for quite a lot of evolutionary developments. Simpler structures can be evaluated and that's what people are debating, with good points being made on both sides.

Rduke55
11-23-2005, 12:20 PM
[ QUOTE ]
Why can't random interactions over an infinite period of time produce what we perceive to be complexity?

[/ QUOTE ]

(I'm agreeing with you noggin, I just am using part of your post to make a point)

This is one of Behe's problems. While variation is random, selection is NOT. This is a major point that a lot of evolution's opponents miss. Stringing together a bunch of amino acids isn't a random process in organisms. The mechanisms have been selected for.

11-23-2005, 12:25 PM
[ QUOTE ]
[ QUOTE ]
Science doesn't provide certainty either. It provides cogency.

[/ QUOTE ]

That's not the point. The point is that you can be certain that nature is ordered because it's axiomatic. If you "knew" through induction, you wouldn't be 100% certain.

[/ QUOTE ]

I'm not 100% certain. And, by "Universe", I'm talking about the whole thing. Not just the part we've observed. We are talking about NotReady, here... and his point was that scientific knowleged relies on faith. Again, science doesn't provide certainty. It uses induction. But, that's not "faith". I have no idea what your argument is here.

Rduke55
11-23-2005, 12:26 PM
[ QUOTE ]
Maybe I will do a thread some day on the objections to the Neo-Darwinian model of evolution

[/ QUOTE ]

I'd like to see this.
There are actual debates on this that have nothing to do with the travesty of intelligent design and Behe's utter crap.
If everyone could get past the God question it might start an actual scienctific debate in this forum rather than what we normally have.

11-23-2005, 12:28 PM
I agree with your preference of thinking of other biological means to explain things. I don't interpret Behe in the way you have described though.
On the Newtonian Physics - it does have limitations. There was a period of time where people realized it had problems explaining certain things. They didnt just say anything that challenges any part of accepted Newtonian physics is a bunch of crap (if they did they were wrong). As a result of examining some of the limitations of Newtonian physics in explaining things, some breakthroughs were made (Relativity). And when we got done we are not throwing out Newton, nor do we have to throw out Darwin.
I'm not quite sure whether we will end up with small tweeks to our theories of evolution or big adjustments. But big adjustments are not outside the realm of my thinking.

11-23-2005, 12:48 PM
A good response to Behe comes from a Catholic: Kenneth Miller (http://en.wikipedia.org/wiki/Kenneth_Miller).

11-23-2005, 01:21 PM
Here's a 20-minute mini-lecture from Miller responding to Behe's irreducible compexity argument:
http://www.meta-library.net/perspevo/preskm-frame.html

jthegreat
11-23-2005, 01:59 PM
[ QUOTE ]
I agree with your preference of thinking of other biological means to explain things. I don't interpret Behe in the way you have described though.
On the Newtonian Physics - it does have limitations. There was a period of time where people realized it had problems explaining certain things. They didnt just say anything that challenges any part of accepted Newtonian physics is a bunch of crap (if they did they were wrong). As a result of examining some of the limitations of Newtonian physics in explaining things, some breakthroughs were made (Relativity). And when we got done we are not throwing out Newton, nor do we have to throw out Darwin.
I'm not quite sure whether we will end up with small tweeks to our theories of evolution or big adjustments. But big adjustments are not outside the realm of my thinking.


[/ QUOTE ]

I think you missed the point of my analogy. Go back and re-read my post.

jthegreat
11-23-2005, 02:02 PM
[ QUOTE ]
I'm not 100% certain. And, by "Universe", I'm talking about the whole thing. Not just the part we've observed. We are talking about NotReady, here... and his point was that scientific knowleged relies on faith. Again, science doesn't provide certainty. It uses induction. But, that's not "faith". I have no idea what your argument is here.


[/ QUOTE ]

No kidding.

You think I'm talking about knowledge gained through scientific means. I'm not. I'm talking about whether or not the universe is ordered, and how certain we are of it. We know it's ordered. If that knowledge is based on induction, we aren't certain, and therefore we aren't certain that science is completely valid, since it's based on the idea of an ordered universe. However, if that knowledge (universe is ordered) is axiomatic, then we *are* certain, and therefore we *are* certain that scientific methods are valid, even if their conclusions aren't 100% true.

NotReady
11-23-2005, 02:18 PM
[ QUOTE ]

Unfortunately it's a false analogy


[/ QUOTE ]

No it isn't.

[ QUOTE ]

First it uses nature as the example of chaos


[/ QUOTE ]

No it doesn't.

[ QUOTE ]

then claims it's perfectly ordered.


[/ QUOTE ]

Yes, because of God's plan.

11-23-2005, 02:23 PM
[ QUOTE ]
[ QUOTE ]
I'm not 100% certain. And, by "Universe", I'm talking about the whole thing. Not just the part we've observed. We are talking about NotReady, here... and his point was that scientific knowleged relies on faith. Again, science doesn't provide certainty. It uses induction. But, that's not "faith". I have no idea what your argument is here.


[/ QUOTE ]

No kidding.

You think I'm talking about knowledge gained through scientific means. I'm not. I'm talking about whether or not the universe is ordered, and how certain we are of it. We know it's ordered. If that knowledge is based on induction, we aren't certain, and therefore we aren't certain that science is completely valid, since it's based on the idea of an ordered universe.

[/ QUOTE ]

That's not true. Validity is not the same as certainty.

[ QUOTE ]
However, if that knowledge (universe is ordered) is axiomatic, then we *are* certain, and therefore we *are* certain that scientific methods are valid, even if their conclusions aren't 100% true.

[/ QUOTE ]

Who is "we"? And who made the axiom? And why does this matter? And... nevermind.

NotReady
11-23-2005, 02:25 PM
[ QUOTE ]

Occam's Razor: "One should not increase, beyond what is necessary, the number of entities required to explain anything."


[/ QUOTE ]

Though I don't believe the Razor is any kind of absolute rule, if it was, it would prove the absolute certainty of God's existence, because what simpler explanation is possible than that one entity (God) explains everything?

NotReady
11-23-2005, 02:31 PM
[ QUOTE ]

If you have the time and incentive, please elaborate on this


[/ QUOTE ]

I simply mean that it's impossible to know much, if anything, with certainty, through either our senses or logical reasoning, yet we must act all the time on incomplete evidence. Partly this is due to our finitude. A large reason is because we cut off the source of knowledge when we rebelled against God. But it goes beyond actions of daily life. Everyone has a fundamental opinion about all of reality, a basic worldview. It may not be well articulated, but the seeds are there in everyone, and one's life is spent filling in the details. For the atheist, the most fundamental presupposition is that he (or mankind in general, or certain wise men) has the right to make the final judgment about what's true or false. And this is a matter of faith, a judgment concerning universal truth that not only can't be proved, but has no supporting evidence.

NotReady
11-23-2005, 02:34 PM
[ QUOTE ]

He's equivocating the word "faith". We've been through all this before. He's saying that induction is "faith" (or at least thinking that induction is valid/reasonable is).


[/ QUOTE ]

No I'm not. I believe the faith required for life is more than just an intellectual admission that one can't prove induction. Everyone has to make some decision about fundamental truth and live by it. This decision can only be made by faith. Even a supposed agnostic who says "I don't know" is making a faith based decision, because he can't know that God can't be known. If he really wasn't sure it would make no sense for him not to spend enormous effort to find out if he's right.

NotReady
11-23-2005, 02:38 PM
[ QUOTE ]

However, as I stated in the thread I started earlier today, that the universe is currently ordered is axiomatic


[/ QUOTE ]

From another thread you said:
[ QUOTE ]

Induction isn't certainty, though. Axioms are.


[/ QUOTE ]

Two problems with these statements:
1. If axoms are certain, and order in the universe is an axiom and therefore certain, why wouldn't induction be certain?

2. A bigger problem is the statement that axioms are certain. If that's so, then God certainly exists, because His existence is an axiom.

NotReady
11-23-2005, 02:40 PM
[ QUOTE ]

Yeah... and we know the universe is ordered by induction. We examine it. Since every oberservation has shown it to be ordered, we induce that it is ordered


[/ QUOTE ]

As Hume proved and I've argued, you are arguing in a circle. The fact that the sun rose today says nothing about whether it will rise tomorrow unless you assume order, then when the sun rises tomorrow you say, "See, order".

11-23-2005, 02:40 PM
I can't leave this alone...

[ QUOTE ]
However, if that knowledge (universe is ordered) is axiomatic, then we *are* certain, and therefore we *are* certain that scientific methods are valid, even if their conclusions aren't 100% true.

[/ QUOTE ]

You sound like NotReady, now. His axiom is that God exists. Yours is that the Universe is ordered.

I'll repeat: I am not 100% certain that the Universe is ordered. I treat it as a fact, however, due to the overwhelming amount of evidence indicating that it is.

Would you mind explaining why you think the universe being ordered has to be an axiom, rather than a scientific fact?

(Note: a scientific fact is not 100% certain. It is considered to be a fact due to the overhwelming supporting evidence, and therefore, by induction is considered to be a fact.)

11-23-2005, 02:46 PM
[ QUOTE ]
[ QUOTE ]

Occam's Razor: "One should not increase, beyond what is necessary, the number of entities required to explain anything."


[/ QUOTE ]

Though I don't believe the Razor is any kind of absolute rule, if it was, it would prove the absolute certainty of God's existence, because what simpler explanation is possible than that one entity (God) explains everything?

[/ QUOTE ]

Actually, God is an "extra" entity. The Universe exists with or without God. "With God" would mean there is one additional entity.

jthegreat
11-23-2005, 02:49 PM
[ QUOTE ]
Two problems with these statements:
1. If axoms are certain, and order in the universe is an axiom and therefore certain, why wouldn't induction be certain?


[/ QUOTE ]

Because knowing the universe is ordered, and knowing perfectly all of the rules/laws involved in that order, are two different things. The former doesn't imply the latter. Like with Newtonian physics. They knew at the time that *some* rules were followed, but that doesn't necessarily mean that they knew precisely what the rules were.

[ QUOTE ]
2. A bigger problem is the statement that axioms are certain. If that's so, then God certainly exists, because His existence is an axiom.

[/ QUOTE ]

An axiom is an idea that is self-evidently true. The existence of a God is not self-evidently true, no matter how much you claim it is. This is the part of the argument where we see a really smart guy say really dumb things.

11-23-2005, 02:51 PM
[ QUOTE ]
For the atheist, the most fundamental presupposition is that he (or mankind in general, or certain wise men) has the right to make the final judgment about what's true or false.

[/ QUOTE ]

You do that as much (if not more, actually) than every atheist. Every time you write on this forum, in fact.

[ QUOTE ]
And this is a matter of faith, a judgment concerning universal truth that not only can't be proved, but has no supporting evidence.

[/ QUOTE ]

There goes that "faith" word again. It's like you want to water it down so much so that when someone talks about your "faith", you can be secure in thinking they also have "faith".

jthegreat
11-23-2005, 02:51 PM
[ QUOTE ]
Would you mind explaining why you think the universe being ordered has to be an axiom, rather than a scientific fact?


[/ QUOTE ]

I did, in the other thread that I guess you don't want to take the time to read.

It's obvious that the universe acts in an orderly way and not chaotic (on a macroscopic scale, at least). The existence of life is only one of example of such an obvious fact. Life would not be possible in chaos.

11-23-2005, 02:52 PM
[ QUOTE ]
[ QUOTE ]

He's equivocating the word "faith". We've been through all this before. He's saying that induction is "faith" (or at least thinking that induction is valid/reasonable is).


[/ QUOTE ]

No I'm not. I believe the faith required for life is more than just an intellectual admission that one can't prove induction. Everyone has to make some decision about fundamental truth and live by it. This decision can only be made by faith. Even a supposed agnostic who says "I don't know" is making a faith based decision, because he can't know that God can't be known. If he really wasn't sure it would make no sense for him not to spend enormous effort to find out if he's right.

[/ QUOTE ]

How much effort have you spent learning every other religion to see if they are right? Not much, I'm sure, since your faith prevents you from finding the truth.

11-23-2005, 02:56 PM
[ QUOTE ]
[ QUOTE ]

Yeah... and we know the universe is ordered by induction. We examine it. Since every oberservation has shown it to be ordered, we induce that it is ordered


[/ QUOTE ]

As Hume proved and I've argued, you are arguing in a circle. The fact that the sun rose today says nothing about whether it will rise tomorrow unless you assume order, then when the sun rises tomorrow you say, "See, order".

[/ QUOTE ]

We've already argued this. You have faith in induction -AND- God. Different types of faith, though. Your faith in induction is supported by rational examination of the world. Your faith in God is supported by your brainwashing throughout the years of reading religious texts, and limiting your environmental exposure to things that may shake that faith.

11-23-2005, 02:59 PM
[ QUOTE ]
[ QUOTE ]
Would you mind explaining why you think the universe being ordered has to be an axiom, rather than a scientific fact?


[/ QUOTE ]

I did, in the other thread that I guess you don't want to take the time to read.

[/ QUOTE ]

I don't know what thread you're talking about. Linky linky.

jthegreat
11-23-2005, 03:00 PM
"On Hume and order in nature". Should be on page 2 by now, hehe.

11-23-2005, 03:04 PM
[ QUOTE ]
An axiom is an idea that is self-evidently true.

[/ QUOTE ]

Why is the universe being ordered "self-evidently true", but god's existence is not?

I would say that the universe's existence is axiomatic. We assume we are not in a dream. But, the fact that it's ordered does not have to be. I'd like to see that other post where you explain why it has to be an axiom rather than a fact.

jthegreat
11-23-2005, 03:06 PM
Okay I've already pointed you to the thread AND, in this thread, restated my argument. Not ordered = chaos. Think about that. Nothing you do, not even your very *existence*, would be possible in chaos.

11-23-2005, 03:06 PM
[ QUOTE ]
"On Hume and order in nature". Should be on page 2 by now, hehe.

[/ QUOTE ]

Ah... this one:
http://forumserver.twoplustwo.com/showflat.php?Cat=0&amp;Number=4003996

Hadn't seen it. I'll read it now.

Jeff V
11-23-2005, 03:06 PM
[ QUOTE ]
--------------------------------------------------------------------------------

Biochemist Michael Behe said the probabliity of linking 100 amino acids to create one protein molecule would be the same as a blindfolded man finding a marked grain of sand in the Sahara desert- not once but 3 different times.


--------------------------------------------------------------------------------



LOL... What a statement.

[/ QUOTE ]

This was not meant to quantify anything.

[ QUOTE ]
But the grossest error committed by the biochemist (? obviously with zero knowledge of statistics or science) is that he fails, by insidiously putting forward an erroneous view pushed by creationists. That somehow, specifics known linking of 100 amino acids are the ONLY way to create a protein molecule useful to life. as they are are something remarkable or peculiar and somehow they only one that could be related to the phenomena of life. Of course, a scientist make no such claim. We have far from exhausted investigating all possible proteins structures (if in themselves they are neccesary to life and there are NO other possibilities. Not known either). To put the analogy a bit more correctly, altough still lacking in quantification, assume that the sand is made up af yellow, black and red grains. Now the probability may be mooted a bit more accurately by saying it would be equivalent to the likehood of the blind man picking three yellow grains in a row if he kept picking grain all his life.


[/ QUOTE ]

That's pretty special- you can debunk a man's life work in one paragraph.

hmkpoker
11-23-2005, 03:27 PM
[ QUOTE ]
[ QUOTE ]

Yeah... and we know the universe is ordered by induction. We examine it. Since every oberservation has shown it to be ordered, we induce that it is ordered


[/ QUOTE ]

As Hume proved and I've argued, you are arguing in a circle. The fact that the sun rose today says nothing about whether it will rise tomorrow unless you assume order, then when the sun rises tomorrow you say, "See, order".

[/ QUOTE ]


No, it's basic conditioning. If each time a certain man gets out of his car he gets a static shock when he touches the door, he will eventually realize that it is going to happen again and will brace himself when he closes it. (this happened to me, because of the shoes I was wearing) Then I wore different shoes, but found myself bracing for the shock. It didn't come. Again and again, I went shock free. I stopped bracing myself.

Humans and animals are very similar in this regard. In my psych experiments we'd lock a rat in a cage where he could press a lever and get food. Eventually he figured it out, and pressed the lever often. Then we disabled the food mechanism, and after a flurry of lever pressing he realized it was worthless and stopped.

The sun is the same way. Let the sun stop coming up a few times, and we won't expect it. This is not circular.

atrifix
11-23-2005, 03:31 PM
[ QUOTE ]
Some people here are crying for quantification and you are right. Youve got to look at the math and the specifics. Just as a rough example - The odds of picking one correct number in a lottery with 4 numbers is one in 10 to the 4th power - or 1 in 10,000. But if I tell you that you have 1,000 numbers that are winners your odds aren't that bad. But what if the lottery has 1 million numbers and you only have 1,000 chances (the corvette, the porshe, the boat and the plane etc...). So whether you have one chance or a thousand chances at a lottery with one million digits doesn't really change the argument of the likelyhood of winning.

[/ QUOTE ]

It doesn't? Your odds of winning at least once with one chance in a million are 1/10^(10^6), your odds of winning at least once with 1,000 chances in a million are 10^3/10^(10^6). Your odds of winning at least once with infinite chances for a finite probability &gt;0 are always at least 1. That is, if there was ANY (finite) probability that the universe/life/etc. would have come together in the way it did, and there was an infinite timeline for that to happen, then it would have happened with certainty.

atrifix
11-23-2005, 03:38 PM
I agree that there are axioms, but the big problem is that no one can ever know with certainty what those axioms are--other than the obvious ones like "there is a world" etc.

I don't understand why existence isn't possible in chaos. It would certainly be different from what we're used to, but there's nothing inherently contradictory about chaos. Why not chaos?

11-23-2005, 04:00 PM
[ QUOTE ]
Okay I've already pointed you to the thread AND, in this thread, restated my argument. Not ordered = chaos. Think about that. Nothing you do, not even your very *existence*, would be possible in chaos.

[/ QUOTE ]

OH. WOW. I wish you would simply have pointed me that that thread to begin with. That's the perfect thread to discuss this idea... and... you started it. So, any response from me on the "axiom propostion" will go in there now. Sorry to hijack the thread. Move on. Nothing to see here.

atrifix
11-23-2005, 04:03 PM
[ QUOTE ]
[ QUOTE ]
This book is written in the conviction that our own existence once presented the greatest of all mysteries, but that it is a mystery no longer because it is solved.

[/ QUOTE ]

I don’t see him fulfilling the promise his all.

[/ QUOTE ]
If you actually interpret this literally, it would be perhaps the greatest book ever written. Presumably he (or whoever wrote the preface) is using some hyperbole and a bit of literary fluff.

11-23-2005, 04:30 PM
[ QUOTE ]
That's pretty special- you can debunk a man's life work in one paragraph.


[/ QUOTE ]

No, Behe did it himself in one sentence, unless he was misquoted or quoted out of context.

jthegreat
11-23-2005, 11:32 PM
[ QUOTE ]
I don't understand why existence isn't possible in chaos. It would certainly be different from what we're used to, but there's nothing inherently contradictory about chaos. Why not chaos?


[/ QUOTE ]

Life requires very specific conditions be continually met in order for it to continue. That's why life cannot exist in chaos.

maurile
11-24-2005, 04:13 AM
[ QUOTE ]
[ QUOTE ]
That's pretty special- you can debunk a man's life work in one paragraph.


[/ QUOTE ]

No, Behe did it himself in one sentence, unless he was misquoted or quoted out of context.

[/ QUOTE ]
The Dover trial on Intelligent Design, in which Behe testified as an expert witness for the defendant school board, is a rich source of material on the emptiness of Behe's views. He was ripped to shreds on cross-examination.

Here's part of the cross examination of Behe (http://www2.ncseweb.org/kvd/trans/2005_1018_day11_pm.pdf).

Ed Brayton's comments on Behe's testimony:

ID Books and Peer Review (http://www.stcynic.com/blog/archives/2005/10/id_books_and_peer_review.php)
Analysis of Behe's Testimony, Part 1: Purpose and Function (http://www.stcynic.com/blog/archives/2005/10/analysis_of_behes_testimony_pa.php)
Behe Disproves Irreducible Complexity (http://www.stcynic.com/blog/archives/2005/10/behe_disproves_irreducible_com.php)
Analysis of Behe's Testimony, Part 4: The Sterility of ID (http://www.stcynic.com/blog/archives/2005/10/analysis_of_behes_testimony_pa_1.php)
Two of Behe's Reviewers Speak Out (http://www.stcynic.com/blog/archives/2005/10/more_on_peer_review_of_behes_b.php)
More on Behe's Testimony (http://www.stcynic.com/blog/archives/2005/11/more_on_behes_testimony.php)

11-24-2005, 05:27 AM
I was feeling bad about my quick jump to conclusions regarding Behe and was about to post a milder version of my comment. I read the entire 136 pages cross-examination. That guy is a joke after all. /images/graemlins/smile.gif

I am sorry I had never heard of the dude. I don't tend to go to populist or vulgarisation works for imformation. But I won't forget it now. This is one guy to definitely on my ignore list when it comes to science or logic. LOL

Jeff V
11-24-2005, 10:32 AM
Care to share an example?

11-24-2005, 10:34 AM
Thanks for the post on Behe's comments in this trial. Behe was not ripped to shreds. He made some excellent points. Among them are:
He encourages students to think critically rather than accept a theory as a fact. Bravo! Exactly the problem with many classes on evolution (and I like the theory of evolution).
His view that statements about "intelligent design" by scientific organizations are political statements is right on the money in my opinion.
The whole discussion of the analogy of intelligent design to archeology ..then SETI...culminating in some poorly worded questions by the cross examiner about cars in his garage was good dialogue from Behe. He is right in his arguments here.
I have a very good friend who does expert witnessing in science (not on this subject). A lawyer has some restrictions but can ask questions that have no basis in fact, they can distort things and twist things. The witness however is under close scrutiny. To see the witness do very well late in the testemony is a sign that he can hold up under fire well. (Sorry I guess I always tend to root for the person thats against the lawyer - I'm like that with whoever is playing the Yankees in baseball too)
I see a lot of comments in this thread tending to dismiss any notion of intelligent design as only an idiot would think such thoughts. Let me just say I respect everyone out there. I usually don't think in terms of trying to make someone else look stupid.

chezlaw
11-24-2005, 10:41 AM
[ QUOTE ]
I see a lot of comments in this thread tending to dismiss any notion of intelligent design as only an idiot would think such thoughts. Let me just say I respect everyone out there. I usually don't think in terms of trying to make someone else look stupid.


[/ QUOTE ]
Its not so much thinking they are silly as thinking they are not being very honest. It could be the case that there is a designer but no-one who has has made even the most cursory study of evolution could honestly argue that complexity is evidence of a designer.

chez

jthegreat
11-24-2005, 12:02 PM
Chips, I guess you didn't re-read my post.

Statements about ID by scientific organizations are generally *not* political, because most of them state some form of "ID is not science.", which is true. It's not and it can't be.

ID is equivalent to saying that since Newtonian physics can't completely explain motion, ghosts must move things around.

11-24-2005, 12:10 PM
Ah OK I am not stupid just dishonest. I agree with evolution as a general principle but think the theory needs work. I also believe that there are some good arguments that show that complex things may show evidence of design. It is good that they did not give me a polygraph test before I got my PhD in Electrical Engineering/Physics. Maybe I was brainwashed as a kid....
This forum seems anomolous to me. I sympathize a lot with atheism as a philosophy. It might well be right in my view. I've met very very very confident atheists before, just wasnt thinking I'd find so many in a poker forum. There's nothing wrong with being confident you are right though - that's fine. But when you get to the point of thinking anyone who disagrees with you is either stupid or dishonest that's an indication that groupthink has taken over for some reason. The reason for that in this particular forum is now what interests me more I think.
Well in any case - best of luck to everyone out there - have a good Thanksgiving all. I dont want to aggrivate anyone really cause I'll listen to anyone's comments on poker. I used to post only in there under a different name a while back but forgot my password.
I suppose I should say that discussing a topic on a forum does show some interest in building one's knowledge so for that kudos to all.

chezlaw
11-24-2005, 12:18 PM
[ QUOTE ]
Ah OK I am not stupid just dishonest. I agree with evolution as a general principle but think the theory needs work. I also believe that there are some good arguments that show that complex things may show evidence of design. It is good that they did not give me a polygraph test before I got my PhD in Electrical Engineering/Physics. Maybe I was brainwashed as a kid....
This forum seems anomolous to me. I sympathize a lot with atheism as a philosophy. It might well be right in my view. I've met very very very confident atheists before, just wasnt thinking I'd find so many in a poker forum. There's nothing wrong with being confident you are right though - that's fine. But when you get to the point of thinking anyone who disagrees with you is either stupid or dishonest that's an indication that groupthink has taken over for some reason. The reason for that in this particular forum is now what interests me more I think.
Well in any case - best of luck to everyone out there - have a good Thanksgiving all. I dont want to aggrivate anyone really cause I'll listen to anyone's comments on poker. I used to post only in there under a different name a while back but forgot my password.
I suppose I should say that discussing a topic on a forum does show some interest in building one's knowledge so for that kudos to all.

[/ QUOTE ]
I'm not saying they are being dishonest (maybe they are being silly /images/graemlins/grin.gif) just that as there exists an extremely simple explanation of how complexity can arise naturally, it is hard not to think they are being dishonest if they use ID as an argument.

chez

11-24-2005, 12:24 PM
Sorry great I didnt agree with your ghosts and goblins point about Newtonian Physics/ design the first time around or the second or the third. I appologize for not confirming with you that I re-read your post. Feel free to post the point again if you like.

jthegreat
11-24-2005, 06:58 PM
Then you're simply being obstinate and ignoring the facts. Proponents of ID see that current evolutionary theory is (supposedly) lacking, but instead of proposing refinements to the theory, they create an untestable, supernatural explanation. It IS equivalent to saying "ghosts move things".

11-24-2005, 08:45 PM
Heya Chips_,

Personally I don't care what proponents of ID think, I don't even want to get in the argument of whether or not it is stupid or dishonest.

What I strongly object to, is to have it introduced in a science education facility. I find that, both stupid and dishonest. Teach it all you want in a religious education facility or a bible study class if you think it matches, but not in a science class.

By the way I have observed many intelligent people holding very stupid notions in specific areas. If they are friends of mine, or I have the time, I will point it out to them to the best of my abilities. It is not a judgement on the whole person, just a remark on one of their idiosyncracies. /images/graemlins/smile.gif

11-24-2005, 09:00 PM
I dont view it as a fact that any hypothesis of design in nature is untestable or that a proposition about ghosts moving furniture would be untestable. I dont view it as a fact that people are arguing specific hypothesis that are untestable. I've heard these arguments for 20 years. I do not accept them as factual arguments. I do not view others who reject this notion as stupid, dishonest or brainwashed. Sorry I keep comming back to that point but its really the superlatives that I see that bother me. The insistance that theories are facts.
Also its perfectly OK in my book to say that we examine certain models with tests and the model did not hold up very well. You don't necessarily have to offer a complete answer as to why it did not hold up well at the same time. When people first did the experiemnents where something was burned and we found that if you added up the weight of all the ashes and smoke, there was more weight than what you started with - that in an of itself is worth examination. You dont have to understand that Oxygen is being added at the same time. That comes later. But the clear realization that previous models of science don't explain a certain phenomenon is not outside the realm of science.
Its funny that if someone were to speculate any specifics about who designed life or why, they would be labelled as preaching religion - "this is not science" would be the cry. So if someone abstains from commenting on who the designer is or their motives, then its deemed "not science" as well.

11-24-2005, 09:15 PM
MidGe - OK fair enough on your last point about idiosyncracies. I think thats a better way of putting it than past posts I have seen.
I agree with you on reservations about the so called "intelligent design movement" leading to changing science curriculums in schools. In some cases "intelligent design" is a cover phrase for introducing religion.
The ideas that I am talking about though really have no place in a bible study. To me this is not about ideas from the bible vs science. Its about looking at current theories and being able to think critically about them.

NotReady
11-24-2005, 09:18 PM
[ QUOTE ]

An axiom is an idea that is self-evidently true. The existence of a God is not self-evidently true, no matter how much you claim it is. This is the part of the argument where we see a really smart guy say really dumb things.


[/ QUOTE ]

If you are writing your own dictionary, then I guess you're correct.

NotReady
11-24-2005, 09:19 PM
[ QUOTE ]

You do that as much (if not more, actually) than every atheist. Every time you write on this forum, in fact.


[/ QUOTE ]

I try to limit my statements concerning the universality and necessity of something to the Bible.

[ QUOTE ]

There goes that "faith" word again. It's l ike you want to water it down so much so that when someone talks about your "faith", you can be secure in thinking they also have "faith".


[/ QUOTE ]

I'm not saying that the faith required for an atheist to do science is equivalent to the faith needed for salvation. But there is a similarity. Beyond that, everyone has a fundamental worldview that they will abandon only for the strongest of reasons, and that worldview is always accepted at least in part by faith. Everyone is convinced about something concerning ultimate truth and no one can demonstrate their position with absolute certainty. Scientists who really believe they are neutral are fooling themselves.

NotReady
11-24-2005, 09:21 PM
[ QUOTE ]

How much effort have you spent learning every other religion to see if they are right? Not much, I'm sure, since your faith prevents you from finding the truth


[/ QUOTE ]

It really doesn't take long to compare the realistic candidates for worldview. The differences between Christianity and everything else is stark. And I have spent much effort verifying Christianity as well as other systems during the process. No human being is complete as to his epistemological awareness during this lifetime. But the heart is set on a path at some point, and the awareness and convictions of people become more firm as life progresses.

NotReady
11-24-2005, 09:22 PM
[ QUOTE ]

Your faith in induction is supported by rational examination of the world.


[/ QUOTE ]

I believe induction is valid because God exists, created the universe, and is reasonable. I see no other possible way induction could be valid.

[ QUOTE ]

Your faith in God is supported by your brainwashing throughout the years of reading religious texts, and limiting your environmental exposure to things that may shake that faith.


[/ QUOTE ]

Sometimes you're just disappointing.

NotReady
11-24-2005, 09:24 PM
[ QUOTE ]

No, it's basic conditioning. If each time a certain man gets out of hi s car he gets a static shock when he touches the door, he will eventually realize that it is going to happen again and will brace himself when he closes it. (this happened to me, because of the shoes I was wearing) Then I wore different shoes, but found myself bracing for the shock. It didn't come. Again and again, I went shock free. I stopped bracing myself.


[/ QUOTE ]

This just describes the process. Hume doesn't argue that. Locke established it very competently. What is lacking is the rational justification.

[ QUOTE ]

The sun is the same way. Let the sun stop coming up a few times, and we won't expect it. This is not circular.


[/ QUOTE ]

What you expect isn't the argument. Hume agrees, you see cause and effect because of repetition. But repetition doesn't amount to rational justification.

NotReady
11-24-2005, 09:25 PM
[ QUOTE ]

I don't understand why existence isn't possible in chaos. It would certainly be different from what we're used to, but there's nothing inherently contradictory about chaos. Why not chaos?


[/ QUOTE ]

There's nothing inherently contradictory about chaos. What is contradictory in chaos is order, meaning, purpose, logic, morality, love, science.

NotReady
11-24-2005, 09:29 PM
[ QUOTE ]

ID is equivalent to saying that since Newtonian physics can't completely explain motion, ghosts must move things around.


[/ QUOTE ]

This isn't any different than saying since design can't completely explain the universe, chance must have done it.

NotReady
11-24-2005, 09:30 PM
[ QUOTE ]

I've met very very very confident atheists before, just wasnt thinking I'd find so many in a poker forum. There's nothing wrong with being confident you are right though - that's fine


[/ QUOTE ]

Don't let them chase you away - if they get too hostile do what I do, ignore them.

NotReady
11-24-2005, 09:32 PM
[ QUOTE ]

What I strongly object to, is to have it introduced in a science education facility. I find that, both stupid and dishonest. Teach it all you want in a religious education facility or a bible study class if you think it matches, but not in a science class.


[/ QUOTE ]

We had a long thread on this idea started by DS some time ago. Stop teaching atheistic evolution, i.e. evolution by chance, and you might find most Christians won't complain too much about the teaching of real biology.

[ QUOTE ]

By the way I have observed many intelligent people holding very stupid notions in specific areas. If they are friends of mine, or I have the time, I will point it out to them to the best of my abilities. It is not a judgement on the whole person, just a remark on one of their idio syncracies.


[/ QUOTE ]

Ditto.

jthegreat
11-24-2005, 09:50 PM
[ QUOTE ]
This isn't any different than saying since design can't completely explain the universe, chance must have done it.



[/ QUOTE ]

Wow, a strawman wrapped in a non sequitur.

What a joke.

jthegreat
11-24-2005, 09:55 PM
[ QUOTE ]
If you are writing your own dictionary, then I guess you're correct.

[/ QUOTE ]

Main Entry: ax·i·om
Pronunciation: 'ak-sE-&amp;m
Function: noun
Etymology: Latin axioma, from Greek axiOma, literally, something worthy, from axioun to think worthy, from axios worth, worthy; akin to Greek agein to weigh, drive -- more at AGENT
1 : a maxim widely accepted on its intrinsic merit
2 : a statement accepted as true as the basis for argument or inference : POSTULATE 1
3 : an established rule or principle or a self-evident truth

Luckily Merriam-Webster's agrees with me.

chezlaw
11-24-2005, 09:58 PM
[ QUOTE ]
We had a long thread on this idea started by DS some time ago. Stop teaching atheistic evolution, i.e. evolution by chance, and you might find most Christians won't complain too much about the teaching of real biology.


[/ QUOTE ]
Does this point to the reality behind the ID argument? Its not that IDers believe ID is science but they want to prevent evolution being taught as science. ID is a tactic.

chez

Lestat
11-24-2005, 10:08 PM
<font color="blue"> Scientists who really believe they are neutral are fooling themselves. </font>

Why isn't it possible to be neutral based on observable evidence?

I consider myself to be neutral (although I'm not a scientist). Now granted, I have to have faith in science, but that is based on observable evidence and has a historic record of making accurate predictions.

Even though you believe as strongly in your convictions as does a scientist, I rarely read anything you write which can be backed up by any observable evidence nor capable of accurate prediction. This is a HUGE difference. You must place tremendous amounts of faith in your non-scientific/non-observable sources.

DougShrapnel
11-24-2005, 10:19 PM
[ QUOTE ]
[ QUOTE ]
We had a long thread on this idea started by DS some time ago. Stop teaching atheistic evolution, i.e. evolution by chance, and you might find most Christians won't complain too much about the teaching of real biology.


[/ QUOTE ]
Does this point to the reality behind the ID argument? Its not that IDers believe ID is science but they want to prevent evolution being taught as science. ID is a tactic.

chez

[/ QUOTE ]Hey Chez and NotReady, I think that I'm begining to argee with NotReady on this. It appears that scientists believe that they have disproven God by evolution and the Big Bang adn the emergance of life. They make statements that these things randomly occured. Science wishes to continue the war that religion started with it. Partaking in this war is the reason why ID is finding an ear in the education community. ID is not ready as a theory to be taught in schools, but since the scientific community will not stop attacking religion, it is nessesary to include a lacking theory, to keep the scientists from an early victory over religion. If we wish to know more about the "random" causes of life and the universe, ID is good for keeping that question current.

11-24-2005, 10:31 PM
Chips_ &amp; NotReady,

Lets try to focus on what the argument is in a simple and abstract way. Let say I have a theory that says that organism evolve from one another, thus a1 turns into a2 which turns into a3..etc. until a100 (the current end of this chain). Now the same thing is theorised to happen for b1 to b100 and c1 to c100 etc,... For some sequences there is strong evidence (Fossil record is complete say for b3,b4,b7 and for e40,e41 to e49, etc..) and the mechanism can be experienced, for a few links at least, in laboratories conditions with fast reproducing organisms. We have now a mechanism and a theory. The theory says that there are 100 intermediate steps between a1 and a100 (it's only a theory) there are records of a1, a11, a21, a31, a41, a51.. to a 91 and a100). There are big gaps, in fact only 10% of the theory is evidenced. But we know that the mechanism is possible for a few links in other classifications. Now, over the next 100 years, we discover many other new fossils, in the correct sequence of geological layers that start confirming what were up till now gaps. We know have 90% of the gaps covered, only for 10% have we found no evidence yet. I would suspect that the gaps left out in no way weaken the validity of the theory.

The theory effectively has been and is being tested...

Note also that we have said nothing about god or lack of god. We haven't even addressed what happen before a1 or before the big bang. We are not concerned with that since we have no way of testing whatever theory we may have. It is outside the realm of science, and quite rightly so.

Now what ID supporters would like to do, is to insinuate the concept of god as an agent for the gaps. This is not science, it cannot be proven and belongs to a completely different domain than science does, similarly to astrology or many other things.

It is totally disingenuous to say that ID is not a religious/political issue. It is a calculated effort at undermining the foundation of what has been the advancement of mankind, science. It is the irrational sphere of mankind trying to usurp the authority of the rational. It should be fought and opposed with all possible vigour to ensure we don't regress to the dark ages of humanity or even worse.

Teach ID wherever, but not in, or as science. That is the point. Science stays out of the way of god/no god issue and religion stays well out of science. Regarding an atheist position rationale. They do exists also. My rejection of the god concept rests in morality and logic issues, not in science. Science makes my postion easier by at least not contradicting it.

I hope this clarifies the argument that I have and explain my criticism of Mr Behe et al... I think they are obscurantists.

Regarding where to teach ID, again, if not in religious and bible classes, I don't know, and I don't care as long as it is not in science.

chezlaw
11-24-2005, 10:32 PM
[ QUOTE ]
[ QUOTE ]
[ QUOTE ]
We had a long thread on this idea started by DS some time ago. Stop teaching atheistic evolution, i.e. evolution by chance, and you might find most Christians won't complain too much about the teaching of real biology.


[/ QUOTE ]
Does this point to the reality behind the ID argument? Its not that IDers believe ID is science but they want to prevent evolution being taught as science. ID is a tactic.

chez

[/ QUOTE ]Hey Chez and NotReady, I think that I'm begining to argee with NotReady on this. It appears that scientists believe that they have disproven God by evolution and the Big Bang adn the emergance of life. They make statements that these things randomly occured. Science wishes to continue the war that religion started with it. Partaking in this war is the reason why ID is finding an ear in the education community. ID is not ready as a theory to be taught in schools, but since the scientific community will not stop attacking religion, it is nessesary to include a lacking theory, to keep the scientists from an early victory over religion. If we wish to know more about the "random" causes of life and the universe, ID is good for keeping that question current.

[/ QUOTE ]
I've no truck with any scientist who claims to have disproven god.

I've also no interest in a war. ID may be a very good tactic, my interests are discovering whether or not it is a tactic or a genuine argument and then doing my best to expose it or argue it on its merits.

chez

college kid
11-24-2005, 10:52 PM
[ QUOTE ]
It could be the case that there is a designer but no-one who has has made even the most cursory study of evolution could honestly argue that complexity is evidence of a designer.

[/ QUOTE ]

Nails must fear you; you just hit one on the head.

college kid
11-24-2005, 10:54 PM
Good post.

college kid
11-24-2005, 11:00 PM
[ QUOTE ]
Stop teaching atheistic evolution, i.e. evolution by chance, and you might find most Christians won't complain too much about the teaching of real biology.


[/ QUOTE ]

Why don't we just abandon the scientific method and ask imaginary fairies how the world works?

Evolution IS science--it is a theory brought about by the continued use of the scientific method, which makes it a science. ID is at the very least NOT science as it does not utilize the scientific method--except in rare cases which either have failed or produced results that can be explained by many means other than an intelligent agent.

That's why it should not be taught in the classroom. It is not science.

DougShrapnel
11-24-2005, 11:16 PM
[ QUOTE ]
I've no truck with any scientist who claims to have disproven god.

I've also no interest in a war. ID may be a very good tactic, my interests are discovering whether or not it is a tactic or a genuine argument and then doing my best to expose it or argue it on its merits.

[/ QUOTE ] Chez, I believe it's nature is more political than scientifical. From looking over the testimony given by Beil?, can't remember who exaclty, There is a huge interest on the probable outcomes of getting ID taught, and not so much on wether ID is actualy correct, or actualy science. What IDers really want is probably much more malicious, and against separation of church and state. ID is the political outcome of a geniune arguement. So yeah it's a tactic.

DougShrapnel
11-24-2005, 11:32 PM
[ QUOTE ]
[ QUOTE ]
I've no truck with any scientist who claims to have disproven god.

I've also no interest in a war. ID may be a very good tactic, my interests are discovering whether or not it is a tactic or a genuine argument and then doing my best to expose it or argue it on its merits.

[/ QUOTE ] Chez, I believe it's nature is more political than scientifical. From looking over the testimony given by Beil?, can't remember who exaclty, There is a huge interest on the probable outcomes of getting ID taught, and not so much on wether ID is actualy correct, or actualy science. What IDers really want is probably much more malicious, and against separation of church and state. ID is the political outcome of a geniune arguement. So yeah it's a tactic.

[/ QUOTE ]After reading what I wrote I'm back to being agaisnt ID. I'll just say it's to close to call from my view, and exit stage left.

NotReady
11-25-2005, 10:13 AM
[ QUOTE ]

...........
2 : a statement accepted as true as the basis for argument or inference : POSTULATE 1
3 : an established rule or principle or a self-evident truth


Luckily Merriam-Webster's agre es with me.


[/ QUOTE ]
2 doesn't. An axiom is a postulate, which means an assumption, which you said an axiom isn't in another post.

Calling something self-evident or an axiom is simply a logical device, the identification of a presupposition for which no proof is offered, or for which proof has already been given. Since order in the universe can't be proved empirically, it can only be an assumption. Simply stating that it is self-evident doesn't make it true, it supplies the presupposition upon which other proofs are based.

NotReady
11-25-2005, 10:15 AM
[ QUOTE ]

Does this point to the reality behind the ID argument? Its not that IDers believe ID is science but they want to prevent evolution being taught as science. ID is a tactic.


[/ QUOTE ]

I can't speak for all IDers. Actually, I'm not one myself in that I'm not a scientist, though I do believe in ID. I have no objection to teaching biology, etc., that is established as science. What CAN'T be established by science is that evolution occurred by chance.

NotReady
11-25-2005, 10:18 AM
[ QUOTE ]

Why isn't it possible to be neutral based on observable evidence?


[/ QUOTE ]

From a Scriputural viewpoint I don't believe anyone is truly neutral. It's possible to represent neutrality in communication and I may not be able to emprically show that someone has said or done something that proves he isn't neutral. It's a religious position. But whether you are neutral or not isn't based on evidence - neutrality concerns the heart attitude.

[ QUOTE ]

I rarely read anything you write which can be backed up by observable evidence or accurate predictions.


[/ QUOTE ]

I don't claim to be a scientist.

[ QUOTE ]

You must place tremendous amounts of faith in your non-scientific/non-observable sources


[/ QUOTE ]

Of course, I've never tried to represent otherwise. I believe that my faith positions have evidence though. It isn't blind faith or a leap in the dark. You basically admit as much yourself. As to accurate predictions, what the Bible does is give an explanation of human nature, history and God that makes sense. No other worldview does this. Strong evidence.

NotReady
11-25-2005, 10:31 AM
[ QUOTE ]

Why don't we just abandon the scientific method


[/ QUOTE ]

Evolution by chance already has.

[ QUOTE ]

that can be explained by many means other than an intelligent agent.


[/ QUOTE ]

The issue isn't whether this or that natural process has an identifiable cause, but whether all natural processes and nature itself has a cause that science can discover - and science claims this at least by inference when it relies on chance through the rubric random mutation. All you have to do to get off the atheist bandwagon and to return to real science is say mutation instead of random mutation. The only way randomness can apply scientifically is through the laws of probability, but random mutation isn't referring to probability law, but to ultimate cause, which is a religious statement, which provokes responses like ID.

[ QUOTE ]

That's why it should not be taught in the classroom. It is not science.


[/ QUOTE ]

If science is defined by predictability explain how you predict what random mutation will do. If you can't then random mutation is UNSCIENTIFIC, and should not be taught in public schools.

11-25-2005, 12:24 PM
MidGe - Thanks for the post. Wow over 3,000 views on this post ! Look what AllinLife - the sushi man - started.
I noticed you adressed both me and NotReady in your post. I think there are a lot of different topics being discussed here. I think he and I are covering different ground on a lot of points.
You make a scientific argument in your first paragraph. On that paragraph --- In considering the evolution of something from A1 to A100 Going from A1 to A2 might require lets say 3 changes in the genetic code. From A2 to A3 another 3 changes ..and so on. Now 6 changes to the genetic code is more than twice as hard as 3. 30 changes are far more than 10 times harder than just 3. You mentioned experiments.....
People have done breeding experiments for centuries. Now with Breeding experiments you are usually selecting variations within the existing gene pool of a species. But still people have always known that you hit barriers. Starting with one genetic code - a few changes is OK - too many changes and you may get a type of animal that cannot reproduce - too many more changes and the animal is far more likely to be dead than alive. A wise man once said that there are far more ways to be dead than alive genetically. Now with evolution - we must realize that on the plus side that we go one step at a time. And that non random selection is in operation. Definately different than making the large number of changes all at once. And also on the plus side (making things easier)there is more than one possible combination of the genetic code that can make an A2. But it turns out from experiments that the fact that we are making more changes to a highly complex and ordered system outweighs the other factors. I'd argue that the math bears this out also. Experiments with micro-organisms follow similar patterns to the breeding experiemnts. Easy to change a few things, very hard to change a medium amount of genes - completly undemostratable to change a large number of genes.

So back to the point you make. If we can clearly see A1 going to A2 going to A3 but then there are gaps...well since the A1 to A2 to A3 progression happened its reasonable to think the gaps would be filled. I would say this is not necessarily true. I think the gap is telling us that the gap area of genetic space if you will is a tough one to live in. Perhaps an impossible one to live in. In may not be simply a case of missing data. In any case saying that if we see part of the A1-A100 sequence it suggests that the whole sequence is likely to have happened - no I can't go along with that as a certainty.
I cant go along with it because of experimental evidence and what I think are mathematical estimates of how evolutionary processes would make major changes to the genetic code.

I have a specific thread that I am thinking of starting on just this topic alone. It will start with a specific example of this concept.

The second part of your post is a continuation of what to me is a philosophical debate. Let me clarify again that by "intelligent design" I'm not refering to a movement. I understand the objection to a movement used as a way of getting religion into the science class. What I refer to when I say design is a possible conclusion based on data. To say that design isnt science in this context says to me that we cross off the list a possible conclusion from the data we collect. And I must also add that the point I make above is not necessarily on its own a case for design. I like the theory of evolution as a general idea.

jthegreat
11-25-2005, 02:58 PM
[ QUOTE ]
If science is defined by predictability explain how you predict what random mutation will do. If you can't then random mutation is UNSCIENTIFIC, and should not be taught in public schools.


[/ QUOTE ]

This is a perfect example of just *how* profound your ignorance of evolutionary theory is.

Ever heard of selection? Random mutations can lead to predictable results when you include the idea of selection.

NotReady
11-25-2005, 04:05 PM
[ QUOTE ]

*how* profound your ignorance


[/ QUOTE ]

This gets tiresome. I'm putting you in with kidluckee and eastbay. Bye.

jthegreat
11-25-2005, 05:10 PM
LOL Jesus, do you have a ton of gall. You dance around issues, argue points that you obviously don't understand, commit all manner of logical fallacies, and then when someone calls you out on any of it, you act all offended.

Well I'm sorry, Princess. Have fun talking to the other not-so-bright people here who can't see through your BS.

Allinlife
11-25-2005, 05:18 PM
Wow.

I would just like to say that before reading the ID theory, I was a firm believer in evolutino theory, but you know what? ID theory just seems to make more sense to me.

I think some people really didn't get the point of the anology, and I think it's absurd that someone could say a wooden chair is more complex than any living organism, for me, the inner workings of a worm seems much more complex than that of inner workings of a car or a 747.

For obvious reasons, both evolution and ID theories have their own flaws/weaknesses. If one theory was so evidently correct over other one, it is rather unlikely to have people around the world to have such a huge debate on creation vs evolution.

Chips, everything you posted pretty much concur with my stand in ID vs evolution.

The book is called "One heart beat away" by the way, its author Mark Cahill. so if you come across the book by any chance, give the book a shot.

I really like what happened in this thread /images/graemlins/smile.gif because most of the people are truly having a intellectual debates rather than your typical internet forum flame war with childish putdowns.

if anyone cares, here is a page from the book, of perhaps a better anology than my original chair anology.

Something from Nothing
Imagine this scenario:
Billions of years ago, a dark substance began to evolve from nothing. It came out of no where. Then there was a big bang. We don't know what caused the bang or why, but it happened.

As many millions of years passed, this substance developed a fizz to it and became sweet. Millions of years later some aluminum molcules formed from nothing, gathered together, and wrapped themsleves around this liquid in the perfect shape of a cylinder. The aluminum then formed a pop top on the cylinder.

Forty or fifty years ago, some red and white paint molecules fell on to the can, forming the words "Coca-Cola", an expriation date, and a complete ingredients lists. Wow-- that is amazing!

This example, given by a friend of mine, describes an absurd way of thinking. It would be an insult to your intellect if I insisted that the above scenario were true. This universe is infinitely more complex than a can of Coke, yet for some reason people are content to believe that it just came out of no where-- that something came from nothing.

But if a Coke can and its contents coulnd't happen by random chance processes, how could something as orderly and intricately designed as our universe have been assembled merely by chance? Logically, we know that's impossible.

jthegreat
11-25-2005, 05:21 PM
Is it impossible for anyone to win the lottery?

And is Coke made of self-replicating molecules?

college kid
11-25-2005, 07:45 PM
Thank you.

college kid
11-25-2005, 08:04 PM
[ QUOTE ]
Something from Nothing
Imagine this scenario:
Billions of years ago, a dark substance began to evolve from nothing. It came out of no where. Then there was a big bang. We don't know what caused the bang or why, but it happened.

As many millions of years passed, this substance developed a fizz to it and became sweet. Millions of years later some aluminum molcules formed from nothing, gathered together, and wrapped themsleves around this liquid in the perfect shape of a cylinder. The aluminum then formed a pop top on the cylinder.

Forty or fifty years ago, some red and white paint molecules fell on to the can, forming the words "Coca-Cola", an expriation date, and a complete ingredients lists. Wow-- that is amazing!

This example, given by a friend of mine, describes an absurd way of thinking. It would be an insult to your intellect if I insisted that the above scenario were true. This universe is infinitely more complex than a can of Coke, yet for some reason people are content to believe that it just came out of no where-- that something came from nothing.

But if a Coke can and its contents coulnd't happen by random chance processes, how could something as orderly and intricately designed as our universe have been assembled merely by chance? Logically, we know that's impossible.

[/ QUOTE ]

I swear to *God* I just debunked this line of thinking a few posts ago. Didn't I?

As per your question about if it was so obviously wrong why would so many people believe in it: PEOPLE ARE STUPID AND STUBBORN. They are able to justify almost insanely illogical things to themselves. Imagine all the astrology believers in the world. Then on top of that, add somebody who is able to spin some kind of false logic that almost sounds reasonable until you pick it apart and you'll see how easy it would be for a person more capable than I am to convince even half way intelligent people that little green elves created the world.

People are stupid, that's why.

college kid
11-25-2005, 08:06 PM
What the hell are you talking about!!??? Do you even know what words come from your mouth or what appears on the screen when you type!? Sorry to be mean and insulting, but... "Wow" is the word that comes to mind.

11-26-2005, 06:03 AM
Hey Chips_,

[ QUOTE ]

I noticed you addressed both me and NotReady in your post. I think there are a lot of different topics being discussed here. I think he and I are covering different ground on a lot of points.


[/ QUOTE ]
Agreed. I though my answer did address some topics that both, or either, of you raised.

[ QUOTE ]

You make a scientific argument in your first paragraph. On that paragraph --- In considering the evolution of something from A1 to A100 Going from A1 to A2 might require lets say 3 changes in the genetic code. From A2 to A3 another 3 changes ..and so on. Now 6 changes to the genetic code is more than twice as hard as 3. 30 changes are far more than 10 times harder than just 3.


[/ QUOTE ]
I was trying to simplify to allow the real issue to come to the fore. You could consider each of a1, a2, a3, etc.. to be intermediate steps. Secondly, even if it was 100 or a thousand, or more times harder, it would not change the argument. In fact, IMO, it would strengthen it.

[ QUOTE ]

You mentioned experiments.....
People have done breeding experiments for centuries. Now with Breeding experiments you are usually selecting variations within the existing gene pool of a species. But still people have always known that you hit barriers. Starting with one genetic code - a few changes is OK - too many changes and you may get a type of animal that cannot reproduce - too many more changes and the animal is far more likely to be dead than alive. A wise man once said that there are far more ways to be dead than alive genetically. Now with evolution - we must realize that on the plus side that we go one step at a time. And that non random selection is in operation. Definitely different than making the large number of changes all at once. And also on the plus side (making things easier)there is more than one possible combination of the genetic code that can make an A2. But it turns out from experiments that the fact that we are making more changes to a highly complex and ordered system outweighs the other factors. I'd argue that the math bears this out also. Experiments with micro-organisms follow similar patterns to the breeding experiments. Easy to change a few things, very hard to change a medium amount of genes - completely undemonstratable to change a large number of genes.


[/ QUOTE ]
Interesting to relate DNA to evolution theory. I think we are just scratching the surface there. We are still very far from understanding the DNA processes and mechanics. I mean, we just managed to map a complete genome recently. We still don't know much about the relationship between genes and features. I hope that your interest may earn you a Nobel prize in the future. Keep it up. I am certain that it will become a very productive branch.

[ QUOTE ]

So back to the point you make. If we can clearly see A1 going to A2 going to A3 but then there are gaps...well since the A1 to A2 to A3 progression happened its reasonable to think the gaps would be filled. I would say this is not necessarily true. I think the gap is telling us that the gap area of genetic space if you will is a tough one to live in. Perhaps an impossible one to live in. In may not be simply a case of missing data. In any case saying that if we see part of the A1-A100 sequence it suggests that the whole sequence is likely to have happened - no I can't go along with that as a certainty.
I cant go along with it because of experimental evidence and what I think are mathematical estimates of how evolutionary processes would make major changes to the genetic code.


[/ QUOTE ]
You are misunderstanding the argument against ID and somewhat falling in the trap that ID'ers would love you to fall in. The test of evolution theory has been going on since it was first formulated, and has uncovered an unbelievable number of confirmations (the discovery of fossil evidence for gaps). Sure there have been adjustments and extensions made to the original theory, but no evidence has been uncovered that sap it in any significant (precisely the ways in which it is objectionable to ID'ers) ways.

[ QUOTE ]

I have a specific thread that I am thinking of starting on just this topic alone. It will start with a specific example of this concept.


[/ QUOTE ]

Go for it dude /images/graemlins/smile.gif

[ QUOTE ]

The second part of your post is a continuation of what to me is a philosophical debate. Let me clarify again that by "intelligent design" I'm not referring to a movement. I understand the objection to a movement used as a way of getting religion into the science class. What I refer to when I say design is a possible conclusion based on data. To say that design isn't science in this context says to me that we cross off the list a possible conclusion from the data we collect. And I must also add that the point I make above is not necessarily on its own a case for design. I like the theory of evolution as a general idea.


[/ QUOTE ]

I am not referring to the movement either, I am referring to the body of work that purports to be science yet fails the fundamental test of science. It cannot be tested. There are a myriad possible ways of explaining gaps, from astrology, to ID and unimagined others. The moment they lend themselves to testing, and confirmation they will be part of science, until then they are NOT and should not be presented as such in an educational context. Where do we draw the line? What bizarre theory do we present as an alternative? Until they are testable and tested they are mere speculation or a parlour game for people that have nothing better to do.

You like the theory of evolution!? I like gravity!? Means very little, what I like, or not, the question debated is: is ID science? The answer to that question is a resounding NO.

I personally don't object to people believing in ID, or astrology, or tarot cards. I don't mind a plethora of books being published about such speculation, and enjoyed and take as fact by some. Just don't confuse it with science, or all you chances at a Nobel prize for science will evaporate.

Enjoy all.

/images/graemlins/smile.gif

11-26-2005, 08:27 AM
MidGe - We had this exchange...

[ QUOTE ]

You make a scientific argument in your first paragraph. On that paragraph --- In considering the evolution of something from A1 to A100 Going from A1 to A2 might require lets say 3 changes in the genetic code. From A2 to A3 another 3 changes ..and so on. Now 6 changes to the genetic code is more than twice as hard as 3. 30 changes are far more than 10 times harder than just 3.


[/ QUOTE ]
[ QUOTE ]
I was trying to simplify to allow the real issue to come to the fore. You could consider each of a1, a2, a3, etc.. to be intermediate steps. Secondly, even if it was 100 or a thousand, or more times harder, it would not change the argument. In fact, IMO, it would strengthen it.

[/ QUOTE ]
-----------------------
1. Of course I'm considering a1,a2 and a3 to be intermediate steps. Thats the whole framework of the discussion.
2. First you said that if we have an evolutionary series A1 to A100 and we observe a few of the steps a1 to a2 to a3 etc - that it makes sense to think the whole series took place. I countered by pointing out - based on huge amounts of data - that going from A1 to A2 is much easier than each succesive step. So the logic that if you observe the easy part, concluding that the hard part must have occured is not necessarily true. Now you are saying that this stengthens your argument if it is 1000 times harder to go from a2 to a3 than it was from a1 to a2. Please explain how this strengthens your argument.

11-26-2005, 08:55 AM
Hiya Chips_,

It strengthen the argument because there is already some evidence of succesive steps, meaning that it is possible...

The gaps are unimportant, maybe they will get filled by evidence maybe not.

11-26-2005, 09:30 AM
MidGe - Sorry for the double post but I wanted to keep this point seperate. As to this discussion......
[ QUOTE ]
Chips:
The second part of your post is a continuation of what to me is a philosophical debate. Let me clarify again that by "intelligent design" I'm not referring to a movement. I understand the objection to a movement used as a way of getting religion into the science class. What I refer to when I say design is a possible conclusion based on data. To say that design isn't science in this context says to me that we cross off the list a possible conclusion from the data we collect. And I must also add that the point I make above is not necessarily on its own a case for design. I like the theory of evolution as a general idea.


[/ QUOTE ]

[ QUOTE ]
MidGe: I am not referring to the movement either, I am referring to the body of work that purports to be science yet fails the fundamental test of science. It cannot be tested. There are a myriad possible ways of explaining gaps, from astrology, to ID and unimagined others. The moment they lend themselves to testing, and confirmation they will be part of science, until then they are NOT and should not be presented as such in an educational context.


[/ QUOTE ]
-----------------------
OK - I'm glad we are not talking about intelligent design as a movement anymore. Let me make my point with an example....

Three scientists walk into a cave that is on the beach, near the sea. At low tide they walk through the wet sand into the cave.
Scientist #1 says "This cave was produced by slow gradual processes - the tide dug this cave over thousands of years by removing sand a little bit at a time and gradually eroding the rock"
Scientist#2 says "This cave was carved by a designer - it looks too perfectly formed to be from just nature"
Scientist #3 gets out a deck of tarot cards.

Now Scientists 1 and 2 go forth into the cave to gather data. Scientist #1 notes discoloration on the bottom of the cave where the tide marks are. And he measures these discolorations, takes some samples, runs some dating tests and comes back with pretty strong case that tides dug out the bottom of the cave.
Scentist #2 looks at the top of the cave and finds chisel marks. He does some tests on the rocks and finds evidence of chisel fragments.
So these two get back together and decide - OK the top part of the cave was carved 1,000 years ago but its twice as deep as it was back then due to water erosion.
Both used scientific techniques to arrive at their conclusions. Scientist #2 isnt going to say that the person that concluded the cave was originally designed can't have made that conclusion through the use of science. He's not going to say "Unless you can bring the original designer here and have him repeat carving this cave you are unscientific" "Your idea can't be tested, can't be repeated etc" "your idea is just like this astrologer here trying to get the answer through tarot cards"
----------------
Now some might say - OK but there are no chisel marks or chisel fragments in nature - others might say yes there are. Still others might say there are things in nature that look like chisel marks and evidence that appears like chisel fragments and were definately not dug by the ocean. I don't think any of these ideas deserve to be placed in the same category as the tarot card reader.

11-26-2005, 10:05 AM
Chips_,

I am getting bored with the same empty arguments repeated ad nauseum.


The chisel work can be reproduced anytime you want. You don't need the original designer/chiseler. If the scientist was saying a dude this did with a tool called "trenhhisgtrtr", and could show no evidence or even explain how a "trenhhisgtrtr" works or what it was, he would have little chance of having is theory accepted, especially if the other scientist could show that water erosion does indeed chisel out rock and could test the hypothesis, at least in part. He doesn't need to repeat the experiment for 1000 years.

The tarot card dude is as good as the "trenhhisgtrtr" dude, but not any better.

If you are going to use an analogy, try to make sure that it is an analogy and that you are not, as ID'ers do, perniciously and insiduously infering things that are not there (in this case a form of anthropomorphic projection on a god concept). If you want/need a god, by all means go for it. Don't push it into science!

I am atheist, based on a moral rationale, not on a scientific one, by the way. But that is not the issue at stake. I don't mind if you tell me that a designer designed evolution... I just don't buy that, but I do not force it on your belief. But in this case the designer concept is not presented as an alternative, without any proof, against evolution, so it is OK.

Science and religion/god belong to two different domains of thought.

NotReady
11-26-2005, 02:17 PM
[ QUOTE ]

I don't mind if you tell me that a designer designed evolution


[/ QUOTE ]

Wouldn't that be Intelligent Design? So the only argument you have with ID is the method?

If so, what happens to your moral rationale for atheism?

11-26-2005, 04:27 PM
[ QUOTE ]
[ QUOTE ]

I don't mind if you tell me that a designer designed evolution


[/ QUOTE ]

Wouldn't that be Intelligent Design? So the only argument you have with ID is the method?

If so, what happens to your moral rationale for atheism?

[/ QUOTE ]

Of course, it would be intelligent design, simply not thaught in science or as science. That is the issue and what is objected to by scientists. Just like tarot, astrology etc... If a school decided to teach astrology, as long as they don't claim it is science I have no problem. There might very well be some interest in the development of astrology. Karl Jung has made an excellent and erudite contribution to the field of psychology by his study of symbolism and within that context astrology may have a value (I would not put Jungian psychology as a science but as part of philosophy, litterature or anothet humanity field). I find it hard to believe how the argument is being twisted over and over again.

It doesn't change my moral rationale/justification for atheism one bit.

NotReady
11-26-2005, 04:36 PM
[ QUOTE ]

Of course, it would be intelligent design, simply not thaught in science or as science.


[/ QUOTE ]

If you agree that evolution could be designed would you also agree that means it can't be by chance? And if it's wrong to teach that evolution is by design would it also be wrong to teach that it's by chance? If one isn't science neither is the other, correct?

11-26-2005, 04:55 PM
[ QUOTE ]
[ QUOTE ]

Of course, it would be intelligent design, simply not thaught in science or as science.


[/ QUOTE ]

If you agree that evolution could be designed would you also agree that means it can't be by chance? And if it's wrong to teach that evolution is by design would it also be wrong to teach that it's by chance? If one isn't science neither is the other, correct?

[/ QUOTE ]

I don't agree that it could but not for scientific reasons.

I totally accept the fact that you or anyone may believe so. In any case, it does not invalidate one iota of the theory of evolution. The designer could have designed a random number generator, or whatever. And I have no objection to a religious class stating that all that his discovered by science is the result of an intelligent agent as long as the religious class is not compulsory ina secular education system. However that statement has no place in a science class. It is not within the purloin of science to speculate about such things, it is a form of charlatanism or "bad" science.

By the way, form my view point, by accepting a god, you simply displace the problem, you don't eliminate it. All the hard questions can now be applied to the god concept (what designed him etc...). If you can accept that he need no designer, why not accept that the world needs none either?

NotReady
11-26-2005, 05:01 PM
[ QUOTE ]

In any case, it does not invalidate one iota of the theory of evolution.


[/ QUOTE ]

You didn't answer the question. Is evolution by chance scientific?

hmkpoker
11-26-2005, 05:05 PM
[ QUOTE ]
[ QUOTE ]

If you have the time and incentive, please elaborate on this


[/ QUOTE ]

I simply mean that it's impossible to know much, if anything, with certainty, through either our senses or logical reasoning, yet we must act all the time on incomplete evidence. Partly this is due to our finitude. A large reason is because we cut off the source of knowledge when we rebelled against God. But it goes beyond actions of daily life. Everyone has a fundamental opinion about all of reality, a basic worldview. It may not be well articulated, but the seeds are there in everyone, and one's life is spent filling in the details. For the atheist, the most fundamental presupposition is that he (or mankind in general, or certain wise men) has the right to make the final judgment about what's true or false. And this is a matter of faith, a judgment concerning universal truth that not only can't be proved, but has no supporting evidence.

[/ QUOTE ]

I meant to respond to this earlier, I hope you get this response...

This is not faith. No reasonable atheist can ever claim 100% certainty about anything. (I think this is the uncertainty principle here, but I don't know) However, we can frequently claim certain things with enough certainty that there is simply no point in questioning it. No one bothers worrying about whether gravity is false and their car will spiral aimlessly up toward the sky. It's a waste of time. To get on with our lives, we must simply assume some things with a sense of confident assumption. Emotions, however, often get the best of us and we may attach to a belief with a fervent tenacity.

No reasonable atheist believes something without a reason, though. Often the belief comes from experiential induction. It can, and frequently does, come from popular say by supposed experts mixed with plausibility. For example, I believe that a fellow named Hunter S. Thompson made a wild trip to Vegas in the sixties, because he wrote about it in his autobiographical book. I wasn't there, people say that's how it happened, it seems plausible and concordant with other data, and there seems no real reason for it to be false. It could be false, but the combination of the improbability and the fact that it doesn't matter if it is makes me not care enough to believe otherwise. It makes sense, so I believe it. I could be proven wrong, however. This is not faith.

Human faith is fallible.

11-26-2005, 05:05 PM
[ QUOTE ]
[ QUOTE ]

In any case, it does not invalidate one iota of the theory of evolution.


[/ QUOTE ]

You didn't answer the question. Is evolution by chance scientific?

[/ QUOTE ]

Yes. And the objection has been answered earlier on this thread (about randomeness not being predictable and therefore not reproducible), if this is what you are trying to raise again.

NotReady
11-26-2005, 05:13 PM
[ QUOTE ]

Yes.


[/ QUOTE ]

I didn't realize this was in the thread before. At any rate, this is the main reason why the controversy exists between Christians and atheists over evolution. If by chance you mean completely undesigned that is a religious, unscientific position. It can't be empirically established.

NotReady
11-26-2005, 05:27 PM
[ QUOTE ]

This is not faith


[/ QUOTE ]

But it is. Faith involves belief and includes action. Humans can neither think nor act without faith.

[ QUOTE ]

No reasonable atheist believes something without a reason, though


[/ QUOTE ]

Nor do reasonable Christians.

jthegreat
11-26-2005, 05:47 PM
[ QUOTE ]
If by chance you mean completely undesigned that is a religious, unscientific position. It can't be empirically established.


[/ QUOTE ]

OF COURSE IT CAN. All you do is take a self-replicating gene, let it reproduce a large number of times, note the mutations and determine whether or not they are a random distribution.

Jeez.

11-26-2005, 05:51 PM
[ QUOTE ]
[ QUOTE ]

Yes.


[/ QUOTE ]

I didn't realize this was in the thread before. At any rate, this is the main reason why the controversy exists between Christians and atheists over evolution. If by chance you mean completely undesigned that is a religious, unscientific position. It can't be empirically established.

[/ QUOTE ]

I mean by chance what the good monk Mendel meant by chance. If you would like to not avail yourself of anything thing available in the world that may have been tainted by the concept of chance, you will be starving for you will only be able to eat fish, veneson and wild berries. Most of today's agricultual and animal food products are the result of the application of the work done by Gregor Mendel.

As far as "completely undesigned", it is indeed a religious position not a scientific one. Scientists are not concerned with the designer or lack thereof, theists are trying to muddy the waters with this concept. Scientists are concerned with models that explain the functionning of what is observable.

And this designer point is a very good examplification of the irreconcialbity and circularity of theist concepts. Why does the world "has" to be desingned, if you can conceive of something else, a god, that need not have been designed? It's not even worth discussing. Let's agree to disagree on this one. It has nothing to do with science and therefore is of little interest to me except in the context of belief as a social phenomena.

My reasons, as I said, to be an atheist are moral ones. Nothing to do with evolution or not.

BluffTHIS!
11-26-2005, 07:56 PM
[ QUOTE ]
Faith involves belief and includes action.

[/ QUOTE ]

So the catholic church is right and salvation is indeed a result of faith and works and Luther and Calvin were wrong. Thus faith alone is indeed dead as scripture says.

hmkpoker
11-26-2005, 08:29 PM
[ QUOTE ]
[ QUOTE ]

This is not faith


[/ QUOTE ]

But it is. Faith involves belief and includes action. Humans can neither think nor act without faith.

[/ QUOTE ]

Would you please supply a definition of "faith" that you would like to use for this discussion? We may have conflicting ideas in mind.

11-26-2005, 11:00 PM
[ QUOTE ]


The chisel work can be reproduced anytime you want. You don't need the original designer/chiseler. If the scientist was saying a dude this did with a tool called "trenhhisgtrtr", and could show no evidence or even explain how a "trenhhisgtrtr" works or what it was, he would have little chance of having is theory accepted, especially if the other scientist could show that water erosion does indeed chisel out rock and could test the hypothesis, at least in part. He doesn't need to repeat the experiment for 1000 years.

The tarot card dude is as good as the "trenhhisgtrtr" dude, but not any better.

[/ QUOTE ]

It seems to me that you are putting an unreasonable burden on the scientist evaluating design. You are saying that unless we can describe HOW life was designed that ANY NOTION or ANY CONSIDERATION of the possiblility of design is unscientific. (I emphasise the any notion phrase just to make my own perspective clear, not to be antagonistic). Maybe some day we could design life ourselves but today we cannot.
You keep going back to insisting that anyone who studies nature and sees any indication of design is equivalent to a tarot card reader and then bring up the objection to teaching religion in the science class. But I thought we agreed before that we are not talking about a movement to teach religion in the science class. (I do see the ten other posts in between ours going after you with religious arguments and you are adressing several perspectives at once.) But the fact that we keep comming full circle back to this does lead me to think that an atheist philosophy is a big driving force here. I would then contend that a fight to preserve an atheist only view of science is as unreasonable as a fight to inject religion into science. By agruing that there is no way you can ever possibly think about design in a scientific context it seems to me like thats whats happening here - this essentially becomes an atheist vs theist debate. What I'm trying to do is claim that scientists can investigate these questions about the origin of life without exluding either possibility from their thinking.
I see in this thread we are now back to Luther, Calvin religion etc. I think its tough here to simultaneously adress what in my mind are seperate discussions. So to be fair to MidGe I do agree with some other points that he is making here.
So my final thought on this thread will be to say that it is possible for scientists to investigate the origin of life or the universe while maintaining the possibility that life or the universe was designed. The notion of design is not so ridiculous that it amounts to spaghetti monsters, tarot card readers etc.
Well I think I'm done on this topic - I will leave you all with the last word. I really wanted AllinLife to get to 10,000 reads on this thread, so I will let the rest of you carry him to that goal.
Thanks for the discussion MidGe - If I had encountered a forum full of religious creationists I might have jumped in on your side. It seems like the atheists are out in full force on this particular forum though so I felt compelled to "bring balance to the force"
Best of luck to everyone out there at the Poker tables - may the River provide you with the mutation you need to advance your hand up the evolutionary ladder to its highest possible form !

maurile
11-26-2005, 11:30 PM
[ QUOTE ]
If you agree that evolution could be designed would you also agree that means it can't be by chance?

[/ QUOTE ]
I think you misphrased that. (If you agree that my bike could be red, would you also agree that means it can't be blue? Of course not. It could be red, or it could be blue.)

But if you're saying that since it's logically possible that mutations are non-random, we shouldn't teach the theory that they are random . . . then, no. That's not correct.

The theory that they are random is testable, and has yet to have been falsified. (The theory that they are designed is not falsifiable. Particular theories about exactly how they are non-random, making specific predictions, are falsifiable -- but all such theories would be falsified very quickly.)

[ QUOTE ]
And if it's wrong to teach that evolution is by design would it also be wrong to teach that it's by chance? If one isn't science neither is the other, correct?

[/ QUOTE ]
No, that's not correct. The theory that mutations are random is falsifiable. If we see a non-random pattern, the theory would be falsified. That's how science works.

The notion that mutations are non-random doesn't make any specific testable predictions, so it is not falsifiable, and is thus not scientific.

11-27-2005, 06:29 AM
Hiya Chips_,

[ QUOTE ]
You are saying that unless we can describe HOW life was designed that ANY NOTION or ANY CONSIDERATION of the possiblility of design is unscientific.

[/ QUOTE ]
Indeed, science is not a record of the ideas that pop up in people's head. They are too many of those, dime a dozen they are. /images/graemlins/smile.gif Science is about postulating, testing, publishing and allowing others to verify the protocol and the results by duplicating.

[ QUOTE ]
this does lead me to think that an atheist philosophy is a big driving force here.

[/ QUOTE ]
What a big change from the real world! /images/graemlins/smile.gif

[ QUOTE ]

I really wanted AllinLife to get to 10,000 reads on this thread,

[/ QUOTE ]
I am still trying, altough it may degenerate and I will then give up. /images/graemlins/smile.gif

[ QUOTE ]
... to advance ... up the evolutionary ladder to its highest possible form ...

[/ QUOTE ]

This shows a misunderstanding of evolution making it a teleological, or moving towards something (perfection), process. Not part of Evolution Theory afaik, just a remnant of theological thinking insinuating itself.

NotReady
11-27-2005, 12:15 PM
[ QUOTE ]

Most of today's agricultual and animal food products are the result of the application of the work done by Gregor Mendel.


[/ QUOTE ]

My only point is that if a scientist is telling me that mutations are completely accidental and uncaused he is making a religious statement. If he is telling me that he applies the laws of probability as I assume is the case with Mendel then he has said nothing about creationism.

NotReady
11-27-2005, 12:20 PM
[ QUOTE ]

Would you please supply a definition of "faith" that you would like to use for this discussion? We may have conflicting ideas in mind.


[/ QUOTE ]

I would define faith as trusting in something or someone and of course includes belief. I don't view faith as having value in itself, it is a necessary condition for action and logical thinking. It is the object of faith that gives merit. And I'm not trying to equate religious faith with the faith a scientist needs to do science or an astronaut needs to step into the ship to the moon. But there are similarities involving the definition I gave. The main point is that faith is needed and is not by definition irrational - and the polemic against Christians is the incessant caterwauling of "irrational" faith as if faith by definition was irrational. The analogy with the FSM and unicorns is espcially vacuous and tiresome. That's why I brought it up and why I draw the similarity between faith in God and other forms of faith.

NotReady
11-27-2005, 12:27 PM
[ QUOTE ]

I think you mis-phrased that. (If you agree that my bike could be red, would you also agree that means it can't be blue?)


[/ QUOTE ]

I phrased it the way I meant. I'm defining chance as the opposite of design. I think you mean it differently.

[ QUOTE ]

The theory that they are random is testable, and has yet to have been falsified.


[/ QUOTE ]

It isn't testable if by random is meant uncaused. It's testable if it's used within the context of probability. Scientists use the word both ways. Used as uncaused it's a religious position. Used as probability it's unobjectionable.

[ QUOTE ]

The notion that mutations are non-random doesn't make a specific testable predictions, so it is not falsifiable, and
is thus not scientific

[/ QUOTE ]

Is it your position that non-random mutations are not falsifiable but random mutations are?

hmkpoker
11-27-2005, 01:02 PM
[ QUOTE ]
[ QUOTE ]

Would you please supply a definition of "faith" that you would like to use for this discussion? We may have conflicting ideas in mind.


[/ QUOTE ]

I would define faith as trusting in something or someone and of course includes belief. I don't view faith as having value in itself, it is a necessary condition for action and logical thinking. It is the object of faith that gives merit. And I'm not trying to equate religious faith with the faith a scientist needs to do science or an astronaut needs to step into the ship to the moon. But there are similarities involving the definition I gave. The main point is that faith is needed and is not by definition irrational - and the polemic against Christians is the incessant caterwauling of "irrational" faith as if faith by definition was irrational. The analogy with the FSM and unicorns is espcially vacuous and tiresome. That's why I brought it up and why I draw the similarity between faith in God and other forms of faith.

[/ QUOTE ]

Ok, I see your similarity. To me, Christian faith seems to have a few properties that distinguish it from (what I'd like to refer to as) secular faith.

1) Secular faith is questionable. When presented with evidence to the contrary, a reasonable person will (should) re-evaluate their beliefs based on the new information. Newton's laws were the cornerstone of physics until Einstein provided us with a new paradigm; accordingly, our understanding of physics had to change, and likely will again.

This is not always the case. There are many times when people refuse to believe something, and it's often related to how much emotional stock is put into the initial belief. A change in belief can be painful. A political activist who is dedicated to a certain cause, when presented with reasonable evidence that his cause is in fact harmful, useless or wrong, will be reluctant at best to abandon it, and will more likely seek ways to reconfirm his convictions so that he can continue fighting for them.



2) Secular faith is not evidence of anything. My belief that the sun will rise tomorrow is not evidence of same; recordings of the phenomenon repeatedly happening are. I get the impression that most Christians believe that their faith is evidence of that which they have faith in. This is wrong. Whether or not you believe that all human reasoning is circular, using one's conviction as evidence for that of which he is convicted is logically incorrect.

From a secular standpoint, the best way to approach truth is to have a sense of responsible detachment from your beliefs. Having secular faith is fine; but when a certain belief is challenged, and something related to that belief causes distress, it is time to re-evaluate. Human reasoning is emotionally influenced, and we often fail at this, but nevertheless, it seems to be the best way to approach the truth...even if we'll never quite get there. Were we to never stray from our secular faiths, society would stagnate and life would succumb to a much poorer quality.

Part of my atheism comes from the fact that God asks us to ignore this fact.

maurile
11-27-2005, 01:22 PM
[ QUOTE ]
[ QUOTE ]

I think you mis-phrased that. (If you agree that my bike could be red, would you also agree that means it can't be blue?)


[/ QUOTE ]

I phrased it the way I meant. I'm defining chance as the opposite of design. I think you mean it differently.

[/ QUOTE ]
Useless digression, but just because something could be A doesn't mean it can't be not-A. I think you misphrased your original statement implying otherwise. Not a big deal; I know what you meant.

[ QUOTE ]
Is it your position that non-random mutations are not falsifiable but random mutations are?

[/ QUOTE ]
Mutations aren't falsifiable. But I know what you mean, and the answer is basically yes.

The idea that mutations are random is falsifiable.

The general idea that mutations are non-random is not falsifiable. Particular ideas about exactly how mutations are non-random (e.g., point mutations involving adenine will always involve a transversion to cytosine) would be falsifiable -- and would be easily falsified.

college kid
11-27-2005, 08:39 PM
I think MANY people would object to ID being taught even if it's not in the science room. School is SUPPOED to be a place where kids learn FACTS and HOW TO THINK RATIONALLY as well as develop a social structure and learn to get along in the world, and maybe even pick up a useful trade.

Teaching ID is just like teaching astrology and neither have any place in public schools.

hmkpoker
11-27-2005, 08:45 PM
[ QUOTE ]
Teaching ID is just like teaching astrology and neither have any place in public schools.

[/ QUOTE ]

If you find a state where the majority are die-hard astrology believers, there will be an issue to teach that in schools too.

maurile
11-27-2005, 09:30 PM
[ QUOTE ]
I think MANY people would object to ID being taught even if it's not in the science room.

[/ QUOTE ]
Not true. It's going to be taught at Kansas University next year in a religious studies course called "Special Topics in Religion: Intelligent Design, Creationism and other Religious Mythologies." The only people objecting are the IDists who insist that ID isn't a religious mythology. LINK (http://www.foxnews.com/story/0,2933,176354,00.html).

college kid
11-27-2005, 10:02 PM
You're right.

I should have rephrased that as I think lots of people would object to calling it anything but a religious topic.

11-27-2005, 10:48 PM
[ QUOTE ]
[ QUOTE ]
I think MANY people would object to ID being taught even if it's not in the science room.

[/ QUOTE ]
Not true. It's going to be taught at Kansas University next year in a religious studies course called "Special Topics in Religion: Intelligent Design, Creationism and other Religious Mythologies." The only people objecting are the IDists who insist that ID isn't a religious mythology. LINK (http://www.foxnews.com/story/0,2933,176354,00.html).

[/ QUOTE ]

nh

Malachii
11-28-2005, 12:15 AM
I haven't read any of the replys, but this argument essentially seems to be the same as William Paley's design argument for why God exists. In short, this argument is a valid one, but it's an inductive argument that can only argue for God's existence with probability. It fails to prove that God must exist or has necessary existence.

Rduke55
11-28-2005, 02:19 PM
[ QUOTE ]


It seems to me that you are putting an unreasonable burden on the scientist evaluating design. You are saying that unless we can describe HOW life was designed that ANY NOTION or ANY CONSIDERATION of the possiblility of design is unscientific.

[/ QUOTE ]

No, the problem is that it's not falsifiable.
It's not a legit theory.

Rduke55
11-28-2005, 02:26 PM
[ QUOTE ]

I'm not a scientist, though I do believe in ID...

...I have no objection to teaching biology, etc., that is established as science. What CAN'T be established by science is that evolution occurred by chance.

[/ QUOTE ]

Why are you an expert on what's established or not established as science if you're not a scientist?

(that sounded harsher than I mean it)

Rduke55
11-28-2005, 02:28 PM
[ QUOTE ]
If you can't then random mutation is UNSCIENTIFIC, and should not be taught in public schools.

[/ QUOTE ]

Again, selection is not random, only the initial variability is (due to the random mutation).

NotReady
11-28-2005, 02:32 PM
[ QUOTE ]

Why are you an expert on what's established or not established as science if you're not a scientist?


[/ QUOTE ]

Because proving that absolute chance exists requires omniscience.

NotReady
11-28-2005, 02:37 PM
[ QUOTE ]

Again, selection is not random, only the initial variability is (due to the random mutation).


[/ QUOTE ]

But selection doesn't decide what the variability will be, it's just a euphemism for the best variable will survive - big deal. You still haven't predicted what the variability will be nor have you proved that chance exists - i.e., unscientific.

NotReady
11-28-2005, 02:41 PM
[ QUOTE ]

No, the problem is that it's not falsifiable.
It's not a legit theory.


[/ QUOTE ]

I don't see how evolution is any more falsifiable than ID.

Or perhaps they are both equally falsifiable.

Rduke55
11-28-2005, 02:49 PM
[ QUOTE ]
[ QUOTE ]

No, the problem is that it's not falsifiable.
It's not a legit theory.


[/ QUOTE ]

I don't see how evolution is any more falsifiable than ID.

Or perhaps they are both equally falsifiable.

[/ QUOTE ]

I think it was Gould that said that finding the fossil of a modern rabbit in the precambrian strata would crush evolution.

Rduke55
11-28-2005, 02:52 PM
[ QUOTE ]
[ QUOTE ]

Again, selection is not random, only the initial variability is (due to the random mutation).


[/ QUOTE ]


But selection doesn't decide what the variability will be, it's just a euphemism for the best variable will survive - big deal. You still haven't predicted what the variability will be nor have you proved that chance exists - i.e., unscientific.

[/ QUOTE ]

You can't predict what the variability will be, it's random. I don't see how you are saying that this is unscientific.
Evolution by natural selection is basically just a mathematical statement. Where it gets the muscle behind it scientifically is the observed evidence of descent from common ancestors.

Rduke55
11-28-2005, 03:15 PM
[ QUOTE ]
[ QUOTE ]

Why are you an expert on what's established or not established as science if you're not a scientist?


[/ QUOTE ]

Because proving that absolute chance exists requires omniscience.

[/ QUOTE ]

So what is science in your opinion?

bocablkr
11-28-2005, 03:15 PM
[ QUOTE ]
MidGe - We had this exchange...


Quote:
--------------------------------------------------------------------------------


You make a scientific argument in your first paragraph. On that paragraph --- In considering the evolution of something from A1 to A100 Going from A1 to A2 might require lets say 3 changes in the genetic code. From A2 to A3 another 3 changes ..and so on. Now 6 changes to the genetic code is more than twice as hard as 3. 30 changes are far more than 10 times harder than just 3.


[/ QUOTE ]

Chips,

Have you ever had a Genetics course? Evolutionary change is not on a logarithmic scale. Who told you that every successive change in the link is harder than the previous one? It could be easier to go from a99 to a100 than from a1 to a2. It is simply dependent on which genes are mutated. It could be one or more. It is not any harder for one gene to mutate in a human now and have some effect than it is in a pre-historic virus.

NotReady
11-28-2005, 04:22 PM
[ QUOTE ]

I think it was Gould that said that finding the fossil of a modern rabbit in the precambrian strata would crush evolution.


[/ QUOTE ]

What about finding all the missing links in human evolution? Wouldn't that crush ID?

Also, as to the rabbit, don't foget HAT (Hopeful Alien Theory).

NotReady
11-28-2005, 04:24 PM
[ QUOTE ]

You can't predict what the variability will be, it's random.


[/ QUOTE ]

I thought to be valid a theory has to make accurate predictions.

NotReady
11-28-2005, 04:27 PM
[ QUOTE ]

So what is science in your opinion?


[/ QUOTE ]

I accept most common formulations of the definition of science, at least I don't find anything objectionable. Post one and I'll comment. But a statement that something is totally uncaused (purely random) is unscientific without omniscience because if there's something you don't know empirically you can't know that what you don't know is the cause.

Rduke55
11-28-2005, 04:31 PM
[ QUOTE ]
[ QUOTE ]

I think it was Gould that said that finding the fossil of a modern rabbit in the precambrian strata would crush evolution.


[/ QUOTE ]

What about finding all the missing links in human evolution? Wouldn't that crush ID?

Also, as to the rabbit, don't foget HAT (Hopeful Alien Theory).

[/ QUOTE ]

So the onus is on evolutionists, that have supporting data, to debunk IDers, that don't have supporting data?
And the fact that the fossil record isn't perfectly complete is not a legit argument for discounting evolution. What about the long spans of time where evolution is clearly shown?

What's the HAT?

Rduke55
11-28-2005, 04:37 PM
[ QUOTE ]
[ QUOTE ]

You can't predict what the variability will be, it's random.


[/ QUOTE ]

I thought to be valid a theory has to make accurate predictions.

[/ QUOTE ]

You keep avoiding the bigger picture. Again, you keep taking only one part of the argument. The initial variability is random, the rest isn't.
And even so, because there's randomness in one part the whole thing is crap even though we can explain how the randomness is involved?
What about diffusion to equilibrium for example? You can't predict with accuracy which molecules are going to go where, but you can predict the result accurately. Is it ghosts moving the molecules?

Rduke55
11-28-2005, 04:40 PM
[ QUOTE ]
[ QUOTE ]

So what is science in your opinion?


[/ QUOTE ]

I accept most common formulations of the definition of science, at least I don't find anything objectionable. Post one and I'll comment. But a statement that something is totally uncaused (purely random) is unscientific without omniscience because if there's something you don't know empirically you can't know that what you don't know is the cause.

[/ QUOTE ]

Errors in transcription are often the cause of mutations. What was the point here?

NotReady
11-28-2005, 04:45 PM
[ QUOTE ]

So the onus is on evolutionists, that have supporting data, to debunk IDers, that don't have supporting data?


[/ QUOTE ]

I'm not going to get into the burden of proof game. That's just an attempt to set the rules of the discussion to favor one side or the other. As to supporting data, that begs the question. IDers claim they have supporting data, evolutionists don't.

[ QUOTE ]

And the fact that the fossil record isn't perfectly complete is not a legit argument for discounting evolution. What about the long spans of time where evolution is clearly shown?


[/ QUOTE ]

If Darwinism predicts something that doesn't show up in the fossil record it certainly is a vaid criticism of the theory. As to evolution being clearly shown, I'm not arguing about other species and whether the evidence is clear or not - I'm arguing human evolution, human fossils, and the lack of intermediaries expected from a Darwinian prediction.

HAT means aliens did it. I used to use this as first announced by the movie 2001 a Space Odessey but have since learned that Crick suggested it even earlier - perhaps Clarke got it from Crick.

NotReady
11-28-2005, 04:47 PM
[ QUOTE ]

You keep avoiding the bigger picture.


[/ QUOTE ]

I've said before I have no objection to using random in a probability sense - like flipping a coin. But it's more often used in the sense of uncaused, which is a religious statement, not a math statement - like the coin flipped itself.

NotReady
11-28-2005, 04:50 PM
[ QUOTE ]

Errors in transcription are often the cause of mutations. What was the point here?


[/ QUOTE ]

I don't know. I got no problem with this statement. Transcription errors occur. Mutations occur. No sweat. Can you cite any IDers who deny this?

Kurn, son of Mogh
11-28-2005, 05:02 PM
That's all well and good, but it's a huge leap from believing that some powerful motive force is responsible for the order in the universe to assuming that force takes an active role or interest in the behavior of one species.

Rduke55
11-28-2005, 05:33 PM
[ QUOTE ]
As to evolution being clearly shown, I'm not arguing about other species and whether the evidence is clear or not - I'm arguing human evolution, human fossils, and the lack of intermediaries expected from a Darwinian prediction.

[/ QUOTE ]

When did ID become just about humans? Behe's favorite example is the flagellum of bacteria.
And there are intermediaries, just not enough to convince people philosophically opposed to evolution of it's truth.

NotReady
11-28-2005, 05:40 PM
[ QUOTE ]

When did ID become just about humans?


[/ QUOTE ]

It's what I'm talking about. Behe can speak for himself.

[ QUOTE ]

And there are intermediaries, just not enough to convince people philosophically opposed to evolution of it's truth.


[/ QUOTE ]

That's the debate of course. I can only say the fossil evidence I've seen purporting to show human descent from primates is less than thin.

bocablkr
11-28-2005, 06:13 PM
[ QUOTE ]
That's the debate of course. I can only say the fossil evidence I've seen purporting to show human descent from primates is less than thin.


[/ QUOTE ]

Been to any good museums of natural history lately?

NotReady
11-28-2005, 06:36 PM
[ QUOTE ]
Been to any good museums of natural history lately?

[/ QUOTE ]

No.

BluffTHIS!
11-28-2005, 07:05 PM
[ QUOTE ]
I can only say the fossil evidence I've seen purporting to show human descent from primates is less than thin.

[/ QUOTE ]

Actually NR, it's clear that you are descended from prehistoric ostriches because you like to stick your head in the sand and ignore science.

Rduke55
11-28-2005, 07:07 PM
[ QUOTE ]
[ QUOTE ]

When did ID become just about humans?


[/ QUOTE ]

It's what I'm talking about. Behe can speak for himself.

[ QUOTE ]

And there are intermediaries, just not enough to convince people philosophically opposed to evolution of it's truth.


[/ QUOTE ]

That's the debate of course. I can only say the fossil evidence I've seen purporting to show human descent from primates is less than thin.

[/ QUOTE ]

How much evidence have you seen? You did say you're not a scientist.

BluffTHIS!
11-28-2005, 07:23 PM
[ QUOTE ]
How much evidence have you seen? You did say you're not a scientist.

[/ QUOTE ]

Choosing to remain in a blissful state of ignorance often allows one to continue to maintain beliefs that otherwise would have to be painfully re-examined. In NR's case this wouldn't mean abandoning Christianity, but only his specific views of it.

NotReady
11-28-2005, 07:39 PM
[ QUOTE ]

How much evidence have you seen? You did say you're not a scientist.


[/ QUOTE ]

I browse the net from time to time. Just wondering, since Darwinism is scientific, and since Darwin himself admitted the fossil record of his day was sorely lacking, has there ever been a precise model done of exactly what the missing links would look like and how many there would be? I mean, since Darwinism is scientific and scientific theories are judged by their capacity to predict.

BluffTHIS!
11-28-2005, 08:31 PM
[ QUOTE ]
and scientific theories are judged by their capacity to predict.

[/ QUOTE ]

Not just predict but also to accurately describe. And you can't get off the hook by arguing about the scientific beliefs and evidence of Darwin's day instead of our own.

maurile
11-28-2005, 10:04 PM
[ QUOTE ]
[ QUOTE ]
[ QUOTE ]

No, the problem is that it's not falsifiable.
It's not a legit theory.


[/ QUOTE ]
I don't see how evolution is any more falsifiable than ID.

Or perhaps they are both equally falsifiable.

[/ QUOTE ]

I think it was Gould that said that finding the fossil of a modern rabbit in the precambrian strata would crush evolution.

[/ QUOTE ]
It was Haldane. (And any rabbit would do.) Good quote, though -- Haldane was absolutely right. There are all kinds of potential falsifications for evolution -- many of them catalogued here (http://www.talkorigins.org/faqs/comdesc/).

maurile
11-28-2005, 10:06 PM
[ QUOTE ]
[ QUOTE ]

I think it was Gould that said that finding the fossil of a modern rabbit in the precambrian strata would crush evolution.


[/ QUOTE ]

What about finding all the missing links in human evolution? Wouldn't that crush ID?

[/ QUOTE ]
Obviously not. We've found thousands of them, and ID is still there.

For any observation that we make, the IDist will just say, "Yeah, the Designer designed it that way." There's no possible obvservation that would be inconsistent with that statement. Which is why ID is unfalsifiable.

maurile
11-28-2005, 10:08 PM
[ QUOTE ]
[ QUOTE ]

You can't predict what the variability will be, it's random.


[/ QUOTE ]

I thought to be valid a theory has to make accurate predictions.

[/ QUOTE ]
Yeah. And the current theory accurately predicts that mutations will be random. Experimental observation are consistent with that prediction, while many other possible observations would be inconsistent with it.

maurile
11-28-2005, 10:09 PM
[ QUOTE ]
[ QUOTE ]

So what is science in your opinion?


[/ QUOTE ]

I accept most common formulations of the definition of science, at least I don't find anything objectionable. Post one and I'll comment. But a statement that something is totally uncaused (purely random) is unscientific without omniscience because if there's something you don't know empirically you can't know that what you don't know is the cause.

[/ QUOTE ]
Don't use "uncaused" as a synonym for "random." Mutations are statistically random -- so says current theory. Nobody thinks they are uncaused.

maurile
11-28-2005, 10:13 PM
[ QUOTE ]
If Darwinism predicts something that doesn't show up in the fossil record it certainly is a vaid criticism of the theory.

[/ QUOTE ]
Correct, but "Darwinism" isn't a theory about how fossils are formed, so it makes no predictions about how many fossils we'll find under what conditions. That's for paleontology and related disciplines to figure out. "Darwinism" -- or, more accurately, evolutionary biology -- does make predictions about what we won't find in the fossil record (e.g., fossil rabits in the precambrian); and all of these predictions are borne out empirically -- so far.

maurile
11-28-2005, 10:14 PM
[ QUOTE ]
[ QUOTE ]

You keep avoiding the bigger picture.


[/ QUOTE ]

I've said before I have no objection to using random in a probability sense - like flipping a coin. But it's more often used in the sense of uncaused, which is a religious statement, not a math statement - like the coin flipped itself.

[/ QUOTE ]
In the context of mutations being random, you're the only one I've seen use "random" as a synonym for "uncaused".

maurile
11-28-2005, 10:17 PM
[ QUOTE ]
I can only say the fossil evidence I've seen purporting to show human descent from primates is less than thin.

[/ QUOTE ]
Prominent hominid fossils (http://www.talkorigins.org/faqs/homs/specimen.html). The fossil evidence is decidedly non-thin.

11-28-2005, 11:36 PM
[ QUOTE ]
[ QUOTE ]
I can only say the fossil evidence I've seen purporting to show human descent from primates is less than thin.

[/ QUOTE ]
Prominent hominid fossils (http://www.talkorigins.org/faqs/homs/specimen.html). The fossil evidence is decidedly non-thin.

[/ QUOTE ]

Very nice link indeed!

I can't help but quoting from a follow-on page from the above link:

"To use an analogy, scientific disputes about exact relationships between hominids might be compared to disputes over how wolves, dogs, coyotes, dingos, hyenas and foxes are related. On that scale, creationists would still be trying to work out how to tell cats and dogs apart."

/images/graemlins/smile.gif

NotReady
11-29-2005, 12:14 PM
[ QUOTE ]

We've found thousands of them, and ID is still there

For any observation that we make, the IDist will just say, "Yeah, the Designer designed it that way." There's no possible obvservation that would be inconsistent with that state ment. Which is why ID is unfalsifiable.


[/ QUOTE ]

There are thousands of gaps in the fossil record, and evolution is still there.

For any observation that we make the evolutionist will say that chance or aliens caused it, or fossils are rare. There's no possible observation that would be inconsistent with that statement. Which is one of the reasons evolution is unfalsifiable.

NotReady
11-29-2005, 12:14 PM
[ QUOTE ]

Don't use "uncaused" as a synonym for "random."


[/ QUOTE ]

I won't if you won't.

NotReady
11-29-2005, 12:16 PM
[ QUOTE ]

Correct, but "Darwinism" isn't a theory about how fossils are formed, so it makes no predictions about how many fossils we'll find under what conditions.


[/ QUOTE ]

I can just say humans existed with dinosaurs but I can't predict how many fossils you will find confirming that.

NotReady
11-29-2005, 12:19 PM
[ QUOTE ]

"To use an analogy, scientific disputes about exact relationships between hominids might be compared to disputes over how wolves, dogs, coyotes, dingos, hyenas and foxes are related. On that scale, creationists would still be trying to work out how to tell cats and dogs apart."


[/ QUOTE ]

No, we would continue to claim that cats don't come from dogs. And we can tell the difference between a monkey and a man.

Rduke55
11-29-2005, 12:21 PM
[ QUOTE ]
[ QUOTE ]

Correct, but "Darwinism" isn't a theory about how fossils are formed, so it makes no predictions about how many fossils we'll find under what conditions.


[/ QUOTE ]

I can just say humans existed with dinosaurs but I can't predict how many fossils you will find confirming that.

[/ QUOTE ]

(humans didn't exist with dinosaurs)

NotReady
11-29-2005, 12:27 PM
[ QUOTE ]

Prominent hominid fossils. The fossil evidence is decidedly non-thin.


[/ QUOTE ]

From the site:

<font color="green">
That is a misleading image, as there are now thousands of hominid fossils.They are however mostly fragmentary, often consisting of single bones or isolated teeth.
</font>

Gimme a break. Please.

I'm going to take these one at a time.

Number one.

<font color="green">
TM 266-01-060-1, "Toumai", Sahelanthropus tchadensis
Discovered by Ahounta Djimdoumalbaye in 2001 in Chad, in the southern Sahara desert. Estimated age is between 6 and 7 million years. This is a mostly complete cranium with a small brain (between 320 and 380 cc). (Brunet et al. 2002, Wood 2002) It has many primitive apelike features, such as the small brainsize, along with others, such as the brow ridges and small canine teeth, which are characteristic of later hominids.
</font>

From another site:

<font color="green">
It's likely that this is a human ancestor. If you ask whether it's absolutely certain that this is a human ancestor my answer would have to be no we are not [sure]," said Bernard Wood of George Washington University
</font>

From Nature magazine:

<font color="green">
Sahelanthropus tchadensis is an enigmatic new Miocene species, whose characteristics are a mix of those of apes and h o m o erectus and which has been proclaimed by Brunet et al. to be the earliest hominid. However, we believe that features of the dentition, face and cranial base that are said to define unique links between this Toumaï specimen and the hominid clade are either not diagnostic or are consequences of biomechanical adaptations. To represent a valid clade, hominids must share unique defining features, and Sahelanthropus does not appear to have been an obligate biped.

We believe that Sahelanthropus was an ape living in an environment that was later inhabited by australopithecines and, like them, it adapted with a powerful masticatory complex. A penecontemporary primate with a perfect and well-developed postcranial adaptation to obligate bipedalism is more likely to have been an early hominid.
</font>

From Talkorigins, definitely not a creationist sympathizer:

<font color="green">
Brunet et al. consider Toumai to be a hominid, that is, on our side of the chimp-human split and therefore more closely related to us than to chimps. This is not at all certain. Some scientists think it probable; others have suggested that it may come from before the point at which hominids separated from chimps, while Brigitte Senut (one of the discoverers of Orrorin tugenensis, "Millennium Man") has suggested that it may be an early GORILLA .
</font> my emphasis

Ok, that's one miss. I'll do the next one tomorrow.

hmkpoker
11-29-2005, 12:35 PM
[ QUOTE ]
[ QUOTE ]

We've found thousands of them, and ID is still there

For any observation that we make, the IDist will just say, "Yeah, the Designer designed it that way." There's no possible obvservation that would be inconsistent with that state ment. Which is why ID is unfalsifiable.


[/ QUOTE ]

There are thousands of gaps in the fossil record, and evolution is still there.

For any observation that we make the evolutionist will say that chance or aliens caused it, or fossils are rare. There's no possible observation that would be inconsistent with that statement. Which is one of the reasons evolution is unfalsifiable.

[/ QUOTE ]


Aliens? Really?

NotReady
11-29-2005, 12:40 PM
[ QUOTE ]

Aliens? Really?


[/ QUOTE ]

Yeah, really. What I call HAT (Hopeful Alien Theory). First (perhaps) proposed by Crick. Popularized by 2001 A Space Odessey. Seriously proposed by serious scientists. Can you spell meteorite?

hmkpoker
11-29-2005, 12:50 PM
[ QUOTE ]
[ QUOTE ]

Aliens? Really?


[/ QUOTE ]

Yeah, really. What I call HAT (Hopeful Alien Theory). First (perhaps) proposed by Crick. Popularized by 2001 A Space Odessey. Seriously proposed by serious scientists. Can you spell meteorite?

[/ QUOTE ]

*blink*

I don't have much to go on here, but I think the vast majority of evolution-supporting biologists don't incorporate HAT into evolution.

NotReady
11-29-2005, 12:59 PM
[ QUOTE ]

I don't have much to go on here, but I think the vast majority of evolution-supporting biologists don't incorporate HAT into evolution.


[/ QUOTE ]

I don't claim they do. I claim they will if necessary which is one reason why evolution is unfalsifiable. Another is punctuated equilibrium, or hopeful monster theory. Already in use to explain the lack of Darwinian gradualism.

Rduke55
11-29-2005, 01:00 PM
[ QUOTE ]
[ QUOTE ]

Aliens? Really?


[/ QUOTE ]

Yeah, really. What I call HAT (Hopeful Alien Theory). First (perhaps) proposed by Crick. Popularized by 2001 A Space Odessey. Seriously proposed by serious scientists. Can you spell meteorite?

[/ QUOTE ]

Does your definition of a "serious scientist" begin and end with their ability to make your points? I could certainly grab Pat Robertson's quotes and generalize them to all religion but that wouldn't be correct would it?

P.S. Alos, you can't count anything Crick did after the double helix Nobel as reasonable examples of scientific thought.

maurile
11-29-2005, 01:49 PM
[ QUOTE ]
There are thousands of gaps in the fossil record

[/ QUOTE ]
And every time a new fossil is found, two new gaps are created.

maurile
11-29-2005, 02:08 PM
[ QUOTE ]
<font color="green">TM 266-01-060-1, "Toumai", Sahelanthropus tchadensis
Discovered by Ahounta Djimdoumalbaye in 2001 in Chad, in the southern Sahara desert. Estimated age is between 6 and 7 million years. This is a mostly complete cranium with a small brain (between 320 and 380 cc). (Brunet et al. 2002, Wood 2002) It has many primitive apelike features, such as the small brainsize, along with others, such as the brow ridges and small canine teeth, which are characteristic of later hominids.
</font>

From another site:

<font color="green">
It's likely that this is a human ancestor. If you ask whether it's absolutely certain that this is a human ancestor my answer would have to be no we are not [sure]," said Bernard Wood of George Washington University
</font>

From Nature magazine:

<font color="green">
Sahelanthropus tchadensis is an enigmatic new Miocene species, whose characteristics are a mix of those of apes and h o m o erectus and which has been proclaimed by Brunet et al. to be the earliest hominid. However, we believe that features of the dentition, face and cranial base that are said to define unique links between this Toumaï specimen and the hominid clade are either not diagnostic or are consequences of biomechanical adaptations. To represent a valid clade, hominids must share unique defining features, and Sahelanthropus does not appear to have been an obligate biped.

We believe that Sahelanthropus was an ape living in an environment that was later inhabited by australopithecines and, like them, it adapted with a powerful masticatory complex. A penecontemporary primate with a perfect and well-developed postcranial adaptation to obligate bipedalism is more likely to have been an early hominid.
</font>

From Talkorigins, definitely not a creationist sympathizer:

<font color="green">
Brunet et al. consider Toumai to be a hominid, that is, on our side of the chimp-human split and therefore more closely related to us than to chimps. This is not at all certain. Some scientists think it probable; others have suggested that it may come from before the point at which hominids separated from chimps, while Brigitte Senut (one of the discoverers of Orrorin tugenensis, "Millennium Man") has suggested that it may be an early GORILLA .
</font> my emphasis

Ok, that's one miss. I'll do the next one tomorrow.

[/ QUOTE ]
Awesome. Thanks for checking that out. So from the above, it appears to be either a late non-hominid ape or an early hominid ape, that could appear in our ancestral lineage just before the most recent common ancestor of chimps and humans, or just after. But either way, it is probably a human ancestor.

The fact that some people consider it to be the earliest hominid and some consider it to be more (non-hominid) ape-like is exactly what you'd expect if there were no sharp dividing line between hominids and non-hominids. ("The world's earliest hominid" is like "the world's tallest midget.") If it has some hominid features and some non-hominid features . . . perhaps it is an intermediate.

Indeed, there are many other good examples of the same phenomenon. In fact, I've got a fun exercise for you if you're up to it. Take a look at this picture and, for each fossil skull, try to categorize it either as a human skull, or a non-human ape skull:

http://www.talkorigins.org/faqs/comdesc/images/hominids2.jpg

I'll start you off with a hint. Fossil A is a modern chimpanzee. Fossil N is a modern human. Which category do the others fall into -- human or non-human?



&lt;I'll wait here while you finish categorizing them . . . &gt;



Here's the significance of that exercise.

The evolutionary model predicts that between chimps and humans there is a smooth progression of ancestral organisms with no sharp breaks in it. You can trace the human lineage backward to the most recent common ancestor of chimps and humans, and then from there trace your way forward to modern chimps, and each step of the way will be just an ever-so-slight modification. There's no sharp dividing line between chimps and humans, just like there's no sharp dividing line between children and adults. The border is blurry.

The creationist model, on the other hand, would predict that there are humans, and there are apes, and there are no intermediates between the two. If you find a skull, it might be an ape skull or it might be a human skull, but it's certainly not in a blurry-border area linking non-human apes to humans.

Take a look at the picture and see what you think. To me, it looks like a pretty smooth transition. The chimp skull (A) is obviously unlike the human skull (N), but in between many of the others really combine features of both, so it's hard to categorize them as "obviously ape" or "obviously human" -- it looks to me like many of them are in between, just as the evolutionary model would predict.

The creationists disagree with that characterization, however. They maintain that all the skulls are either clearly ape or clearly human. They are separate categories with no blurring in between, and any given skull fits into one category or the other.

The curious thing, however, is that while creationists all agree that each skull is either "clearly ape" or "clearly human," they disagree about which category the particular skulls go into.

Look at skull D. Creationists pretty much agree that it is a non-human ape skull. So far so good.

But now look at skull F. Three creationist sources say that it's clearly a non-human ape skull. Eight creationist sources, however, identify it as being merely a human skull.

Same goes for various other skulls. Of those same 11 creationist sources, eight list Java man as being a non-human ape while three list it as being human (a different 8-3 breakdown from the previous skull, however). Meanwhile, six list Peking man as being a non-human ape, while seven list it as being human. Skull H is reported to be a non-human ape by one source while twelve list it as being human. And so on.

So you see, there are plenty of links betwen apes and man. It's just that the creationists refuse to see them as links, but instead see them as being completely ape or completely human.

It's as if I said that there was a smooth transition between black and white, with various shades of gray in between. But every time I find a new shade of gray, the creationists all claim that it's not really gray; it's just black -- or white -- one or the other, only they can't agree among themselves on which it is.

<font color="#000000">A</font><font color="#333333">B</font><font color="#666666">C</font><font color="#999999">D</font><font color="#C0C0C0">E</font><font color="#CCCCCC">F</font><font color="#FFFFFF">G</font>

A is clearly black while G is clearly white . . . but what about D? Creationists would all agree that there's no such thing as gray, so it's obviously either completely black or completely white . . . but 65% think it's black while 35% think it's white.

That's what the transitional hominid fossils are like. They are transitional between apes and humans. But since creationists aren't allowed to believe in transitional fossils, they have to categorize them as being completely ape or completely human . . . but they can't agree on which are which.

NotReady
11-29-2005, 02:12 PM
[ QUOTE ]

Does your definition of a "serious scientist" begin and end with their ability to make your points?


[/ QUOTE ]

No.

[ QUOTE ]

I could certainly grab Pat Robertson's quotes and generalize them to all religion but that wouldn't be correct would it?


[/ QUOTE ]

When he is correct, which he often is, yes. I'm willing to discuss any theological statement he makes rather than just calling him a kook, but you want to dismiss any scientist who questions orthodox evolution as heretical by definition.

I did a short google of the alien thing. About 6 months ago I got more hits than I did this time which perhaps means the theory is receding. Here are some I found:

http://www.sciencedaily.com/releases/2001/08/010824081046.htm

http://www.comdig.org/index.php?id_issue=2000.12#440

http://www.brightsurf.com/news/feb_04/EDU_news_022004_c.php


[ QUOTE ]

P.S. Alos, you can't count anything Crick did after the double helix Nobel as reasonable examples of scientific thought.


[/ QUOTE ]

If a Nobel prize winner isn't a serious scientist, who the heck is?

maurile
11-29-2005, 02:18 PM
The link between humans and non-humans is not only supported by fossil evidence; the DNA evidence is also overwhelming.

This is taken mostly from here (http://www.gate.net/~rwms/hum_ape_chrom.html).

Some time ago, a few scientists were examining chimpanzee sperm in their laboratory (don't ask how it got there), and noticed that it had 24 chromosomes. Meanwhile, they knew that human sperm had only 23 chromosomes.

What could account for the difference? They theorized that the common ancestor for chimps and humans may have had 24 chromosomes, but sometime after the human and chimp lineages had branched off from each other, two of the humans' chromosomes had fused together.

Ah, a testable theory!

The theory makes several predictions:

1. One of our chromosomes would look like two chimp chromosomes stuck together.

2. This same chromosome would have an extra sequence in it that looked like a centromere. (Centromeres are things in the middle of the chromosome that microtubules grab onto to divide a pair of chromosomes during mitosis.)

3. It would also have telomeres (ends), but in the middle, and they would be in reverse order. Sort of like this:

ENDchromosomestuffDNEENDchromosomestuffDNE

The "DNEEND" in the middle is what two telomeres would look like if two chromosomes were stuck together.

These predictions were all made before the answers to them were known. Then they were tested. The results?

See image. (H = human, C = chimp, G = gorilla, O = orangutan.) (http://www.gate.net/~rwms/hum_ape_chrom_2.gif)

1. "Look like" is inexact, so you don't have to count it as a testable prediction. See the similarity for yourself, however, before moving to the more definite predictions.

2. The human Chromosome 2 -- the one that looks like two chimp chromosomes fused together -- does indeed have an extra sequence in it that is characteristic of a centromere. To quote the abstract (http://www.gate.net/~rwms/hum_ape_chrom.html#4) from the relevant research paper, "In situ hybridization, under low stringency conditions with two alphoid DNA probes (pY alpha 1 and p82H) labeled with digoxigenin-dUTP, decorated all the centromeres of the human karyotype. However, signals were also detected on the long arm of chromosome 2 at approximately q21.3-q22.1. Since it is supposed that human chromosome 2 originated by the telomeric fusion of two ancestral primate chromosomes, these findings indicate that not only the telomeric sequences, but also the ancestral centromere (or at least its alphoid sequences), have been conserved."

3. The human Chromosome 2 also has a sequence characteristic of a telomere (the "DNEEND" analogue) exactly where it was expected to be found. To quote the abstract (http://www.gate.net/~rwms/hum_ape_chrom.html#3) from the relevant research paper, "We have identified two allelic genomic cosmids from human chromosome 2, c8.1 and c29B, each containing two inverted arrays of the vertebrate telomeric repeat in a head-to-head arrangement, 5'(TTAGGG)n-(CCCTAA)m3'. Sequences flanking this telomeric repeat are characteristic of present-day human pretelomeres. BAL-31 nuclease experiments with yeast artificial chromosome clones of human telomeres and fluorescence in situ hybridization reveal that sequences flanking these inverted repeats hybridize both to band 2q13 and to different, but overlapping, subsets of human chromosome ends. We conclude that the locus cloned in cosmids c8.1 and c29B is the relic of an ancient telomere-telomere fusion and marks the point at which two ancestral ape chromosomes fused to give rise to human chromosome 2."

These second and third predictions would have been extremely unlikely to be confirmed if the theory that generated them -- that a chromosomal fusing occurred in the human lineage after it branched off from the chimpanzee lineage -- were not correct. If chimps and humans were always separate, and do not share a common ancestor, then why is there evidence of an ancestral fusing? If God wanted a single long chromosome, he could just make a single long chromosome. He wouldn’t have to do it by actually fusing two chromosomes and leaving the telomeres there.

The telomeric remnant is like the human coccyx (http://www.freewebs.com/oolon/SMOGGM.htm#coccyx) at the level of our DNA. And the argument that it might have been put there on purpose by a designer is like the "God planted fake dinosaur bones deep in the earth to give the false appearance of an evolutionary past" argument at the level of our DNA.

maurile
11-29-2005, 02:22 PM
There is also the matter of our vitamin C pseudo-genes: another smoking gun.

Mammals are generally able to synthesize their own vitamin C. Humans, however, are unable to do so. We must get vitamin C from our diets or else we will get scurvy. We have all of the genetic mechanisms required to synthesize vitamin C, but it appears that a single mutation has "broken" this mechanism, rendering it ineffective.

As it turns out, gorillas, chimpanzees, orangutans, and in fact all primates are in the same boat as us. They all have the general mammalian genetic mechanisms for synthesizing vitamin C, but in each case the mechanism is broken.

Here's the interesting part.

There are probably a billion different ways to break the genetic mechanism used to synthesize vitamin C. Mutations are all essentially random, so all of these billion mechanisms are equally probable. The "smoking gun" is that all primates (including humans) have the exact same deleterious mutation in their vitamin C synthesizing genes, in the exact same location. The odds against this occurring by random mutation separately in all primates (and only primates) are astronomical.

On the other hand, we know for a fact that DNA is inherited, and that mutations are passed down to descendents.

So the two obvious possibilities are that (a) the exact same mutation occured in each species of primate independently, or (b) the mutation occured once in a common ancestor of all primates and was subsequently inherited by all its descendents, including chimps, gorillas, and humans.

Typos in written English are a decent analogy for genetic mutations. Suppose you stumble upon a KJV Bible that, in Mark 7:27, says, "Go and sin on more" (instead of "no more"). Now suppose that you find that same typo in many different printings of the Bible. (This actually happened, by the way. You can also find some KJV editions that command, "Thou shalt commit adultery.")

What's more likely? That each printing was from a different manuscript that independently made the same error? Or that the error was made once in a single manuscript and was subsequently copied numerous times?

Obviously, the latter is more likely -- in both the KJV and in primates' vitamin C synthesizing genes. The fact that we have the exact same deleterious mutation in the exact same place is exceptionally strong evidence that the vitamin C pseudogenes in chimps and humans were copied from the same source.

BTW, guinea pigs also have a broken vitamin C mechanism. Theirs, however, is broken in a completely different place, showing that it is almost certainly unrelated to the primate break.

hmkpoker
11-29-2005, 02:51 PM
[ QUOTE ]
[ QUOTE ]

I don't have much to go on here, but I think the vast majority of evolution-supporting biologists don't incorporate HAT into evolution.


[/ QUOTE ]

I don't claim they do. I claim they will if necessary which is one reason why evolution is unfalsifiable. Another is punctuated equilibrium, or hopeful monster theory. Already in use to explain the lack of Darwinian gradualism.

[/ QUOTE ]

But evolution has nothing to do with aliens. If genetic changes between species are the result of alien intervention and not genetic mutation, then evolution would be falsified.

NotReady
11-29-2005, 03:07 PM
[ QUOTE ]

But evolution has nothing to do with aliens.


[/ QUOTE ]

It's generally more to do with abiogenesis but it can also be used to replace early evolution. The idea would be that life initiated elsewhere, some evolution occurred, those early life forms migrated here (meteorites perhaps) and then evolution continued here as advertised. The whole idea is to get around the perceived inadequacy of 4.5 billion years to account for today's biological facts. I saw some of this when examining the application of the laws of probability to abiogensis and evolution. There are models that show 4.5 billion years isn't nearly enough time, so HAT allows a few extra billion years.

hmkpoker
11-29-2005, 03:29 PM
[ QUOTE ]
It's generally more to do with abiogenesis but it can also be used to replace early evolution. The idea would be that life initiated elsewhere, some evolution occurred, those early life forms migrated here (meteorites perhaps) and then evolution continued here as advertised. The whole idea is to get around the perceived inadequacy of 4.5 billion years to account for today's biological facts. I saw some of this when examining the application of the laws of probability to abiogensis and evolution. There are models that show 4.5 billion years isn't nearly enough time, so HAT allows a few extra billion years.

[/ QUOTE ]

**I should note at this time that, lacking any study in biology or anthro, I feel like a retard defending evolution, as I have nothing to go on but philosophical ground. I digress**

To me, this doesn't sound like an attempt to prove or disprove evolution, it sounds like an attempt to explain our origins based on the assumption that evolution is true.

Let's simplify things a little. We don't need to know every chromosome, missing link, ancient species and carbon atom to believe that evolution is true. Evolution revolves around one basic principle, as I see it:

Genetics data is prone to mutation. Accordingly, slight mutations within the parents' offspring will often occur.

Assuming that this is true, the rest of the theory follows logically: some mutations make survival more likely than others. Those with advantageous mutations are more likely to pass on their genes, and natural selection takes place.

Disprove mutation, and you disprove evolution. Would you agree with this statement?

Lestat
11-29-2005, 04:04 PM
NotReady-

I am fascinated by the fact that you can know all this stuff, seem to have even studied all this stuff, have the intelligence to understand all this stuff, yet still manage to be a Christian fundamentalist! ??

Seriously, I'm intriqued. It makes me think I might've missed something.

bocablkr
11-29-2005, 04:05 PM
Maurile,

No matter how much 'proof' you present (and you did an excellent job in those two posts) some people will never get it. You mentioned so many interesting and undeniable facts that you wonder how anyone could dispute evolution but NotReady will never be convinced.

When you throw in some common facts like you can't distinguish most mammals from each other early on in their embryonic states, the fact that we have the remnants of a tail or an appendix, that dolphins have lungs, ad infinitum you would think everyone would believe. But alas, I think we are wasting our time.